Open navigation menu
Close suggestions
Search
Search
en
Change Language
Upload
Sign in
Sign in
Download free for days
50%
(2)
50% found this document useful (2 votes)
225 views
392 pages
Aops Prealgebra Chapter7-15
Aops Prealgebra Chapter7-15
Uploaded by
evanzkong
Copyright
© © All Rights Reserved
We take content rights seriously. If you suspect this is your content,
claim it here
.
Available Formats
Download as PDF or read online on Scribd
Download
Save
Save Aops Prealgebra Chapter7-15 For Later
Share
50%
50% found this document useful, undefined
50%
, undefined
Print
Embed
Report
50%
(2)
50% found this document useful (2 votes)
225 views
392 pages
Aops Prealgebra Chapter7-15
Aops Prealgebra Chapter7-15
Uploaded by
evanzkong
Copyright
© © All Rights Reserved
We take content rights seriously. If you suspect this is your content,
claim it here
.
Available Formats
Download as PDF or read online on Scribd
Carousel Previous
Carousel Next
Download
Save
Save Aops Prealgebra Chapter7-15 For Later
Share
50%
50% found this document useful, undefined
50%
, undefined
Print
Embed
Report
Download
Save Aops Prealgebra Chapter7-15 For Later
You are on page 1
/ 392
Search
Fullscreen
Solution De Re D2 7x (B+3+7) Tx (4-457) (13-7) x 7-18 7x (241047) 1 conta to anne wn my tes aang ruc MCU tons CEA. aries ath oer 0 meh coe eee mush es, eng ene ‘ie stern omy hours ate ~ Agata oie CHAPTER 7 1 Pegi Conversions, and Rates Inthis chapter we vil lscuss: 1 ratios that compate ta or more quantities 1 conversion factor that ve use to convert measurements fom one unto another (for example rom Inches to yards), nd * rates that measure how » quantity changes overtime “These ideas reall elated-in fact the last tw (conversions and rates} can be thought of as special cases of ratio. 7.1 What is a Ra ‘A ratio is used to compare the relative quantities of (usually) two groups or terns of data ‘A simple example should give you an idea of what we mean. Suppose @ certain sclence class hes 10 gts and 7 boys. We would say that ‘the rato of girl to boys in the class ie 100 7. We can write this in a few efferent ways: 1Wt7, 10:7, 10/7 ‘The notation 10 : 71s the most commonly used when wing, and is usualy spoken “1010 7" ‘The key concept to remember is that the ratio only compares the twa quanttieit doesnt tell us anything about the amount of the ‘quantties. For example, suppose that you know that in a history class, the ratio of gis to boys Is 2:3, All this tells you ls that for every 2 ‘i, there are 3 boys. There might be 2 gis and 3 boys inthe clas, or there might be 10 gis and 15 boys in the clas, or there might be 200 gts and 300 boys in the class (in avery large classroom) All you know is that f there are On gil, then there are Bn boys, but you ont know what ns, Concept: Aralio gives aelative comparison of two quantities tt doesnt tell you anything about a the total amount ofthe quantities, Ratios behave aft like fractions. They re usualy writen in simplest form asa rato of two postive integers wth no commen facto larger than 1. To change our example, yppose that vere now considering a math clase with 12 gla and 6 boys, We cauk! write that the ratio of ‘ils to boys is 1210 6 or 12 : 6, However, we could also divide the studente into 6 identical groups, where each group has 2 gitls and 1 boy. This means thatthe ratio of gil to boys is 2: 1 in each group, but since all the groups are the same, the overall ratio of gis to boys Is also : 1 We thereforehave 12: 6 = 2:1. ‘We ypealy reduce to simplest form by dvisng the greatest common factor from each part ofthe ratio, So to continue our example, since 2 6 ° the greatest common factor of 12 and 6 Is 6, the ratio of gis to boys Is (F210 1, This also makes sence If we use fraction notation: 12/6 = 9/1. In words this means that for every 2 gis there Is 1 boy. This process of reducing the ratio fo simplest frm is also called simplifying the rato, Definition: To simplify a ratio means to write It as a ratio of integers with no commen factor larger than 1 Problems. [roten 74 @ kampioSokton | [smote towing ato |(@ 2:10 () 9:6 @ © Problem 7.2 lump io Solution ‘The atio of eats to dage in a pet chop ie 2:5. If there are 25 dogs inthe shop, then how many cats ae there? Problem 7.3 {© Jump t Solution | Mrs. Mile claes has rato of grist boys of 4:3. there are 35 students ithe clas, then how many of them are gis? Problem 7.4 Source: MATHCOUNTS. © Jump to Selution ‘A. 10-foot length of rope is cut into two pleces whose lengths are in the ratio : 4. What s the length ofthe longer pleve? Problem 7.5 {© Jump to Solution My aunt's candy jarhas 56 pleces of candy. Se only has butterscotch and jelly beans, and the ratio of butterscotch to ely beans Is {ike jelly beans more, and I want to ad some jelly beans 20 thatthe ratio of butterscotch to jelly beans is 2: 1. How many jelly beans shoul lade? Paral Seo ee ‘Solution fr Problem 7.1 (2) The greatest common factor of 2 and 1Die 2. So, we divide both parte of the ratio by 2, and we get 2.0 2a 52a 1b, ‘ 201 Thetfore the sinplfed aos 1:5. We an also see this using aeons since => = (6) The greatest common factor of 9 and 6 Is 3, 60 we divide both pats ofthe ratio by 3, and we get 96 926-525 = 5:2 ‘As fractions, this isthe a factions sis the same es 2 = 1a (©) Towne 5 : 5 a8 a ratio of postive integers, we need to multiply both parts of the rao by some number that wll cancel both ‘denominators. The number we need is the least common multiple of the two denominators, which Is 6. This gives us 5a-( 23 ‘Asa fraction, this simplfiction is equivalent(@) tt is usually easier to work with mixed numbers by first converting them to fractions. So we start by writing ghagg 8,1 Tt yy to? 13 eI eae eg at aio “Ten. mutiphing by 9 wl remove he denominators, hing us 1 alta (7 234 pag This is aati of two postive integers with no common factor (othe than 1) s0 were done (©) Wecan frst wite the ratio asa ratio of integers by multiplying by 10 14:24 = 14: 24 “Then, 14 and 24have reatest common factor 2, so we vide by 2to finish the simplification: 424 La: 24 = 14:24 erry Set te ete dein ‘Solution for Problem 7.2: The ratio 2: 5 means that for every 2 cats, there are 5 dogs. Naturally thls also means that for every § dogs, there are 2 cats. The latter way of thinking about thi ratio seems more useful, since we are told how many dogs there are and we vant fo figure out how many cats there are. There are 25 dogs Inthe shop. We can think of this as § groups of 5 dogs each, Each group of S dogs has a coresponding group of 2 cats Solin the shop, there are 5 groups of 2 cats each, for a total of 5-2 = Weate, ‘Another way to think about this problem Isto let cbe the number of cats in the shop, Concent: Since the ratio of eats to dogs inthe shopis 2:5, we have the equation 5 = 2:5. ‘We can make the second ratio look ike the frst one by muitislying the parts ofthe second ratio by 5.aiving 6:95 = 2:5 = (2-5) 25-5) = 1035. CCompating the first and last ratios above tells us that ¢ = 10,0 there are 10 cate in the shop. Indeed, we can check that 10: 25 = 2: 5 Setting two ratios equa, such as ¢ Section 7.3. a Pi ‘Solution for Problem 7.3: At frat, t saems like we may not have enaugh information to cove this pralem~the ratio tlle usa relationship between gis and boys, but we're not given the numberof gis or the numberof boys, Instead, we're just told the total number of students, However, ve can use the gven rato to construct anew ratio: the ratio of gist the total number of students. We know thatthe 4 : 3 gis to-boys ratio means that for every 4 gis there are 3 boys, So this means that for every 4 gis, there are 4-4-3 = 7 total students (girlsLety be the numberof gis inthe class. Since there are 35 students inthe class, we have Mattpying the parts ofthe fist ato by 5 gives 80g = 2nd there are 20 gis a4 3 ‘Another way to think of his that girls make up —* — 3 of the total numberof students, and boys make up — >. = 3 of the total v nmin 3" oy Pass numberof students. Asa check notice that 5 + 5 — 7 — 1 soils and boys together makeup al ofthe students Important: Suppose we are using a ratio to compare Two quantties that together make up & ° group (auch as girls and boye ina class). IF the two quantities are inthe ratio a: b then the frst quantity makes up —" ofthe whole, andthe second quantity makes ayo up of the whole. Paso" Try using this method inthe next problem: Problam 7.4 ne aed nee eee Serene ‘Solution for Problem 7.4: When we cut the rope, it will consist of two pleces that together make up the whole rope. Since the pleces have 11 44 lengths in ratio 1: 4, the shorter piece will make up > = 2 of tne origina length andthe longer plece will make up —*— = 4 of lengths in ratio 1 : 4 the shorter piece wll make up =; = = ofthe orginal length arth longer lece ll makeup =; = ofthe 4 cxiginal length The longer plece is = of a 10Joot rope, so ts lengths $ (a0 oa) = ($10) fat = 8 ft, jos -Pats and Wnoles Problem 7.5 My aunts candy jar has 56 pleces of candy. Se only has butterscotch and jelly beans, and the ratio of butterscotch to elly beans is 5: 2. ire een eaten ers eet tees ges Perrier ‘Solution for Problem 7.5; We start by figuring cut how many pleces of each type of candy are inthe Jar. For practice, well show the two main methods. ‘Method 1: Use a variable. Since the ratio of burterscotch to jelly beans Is 5: 2, we know that thee are fin pleces of butterscotch and 2. Jelly beans for some n. We also know that there are 56 pieces total. So On + 2n = 56, which means 7r = 56. Dividing by 7 gives us = 8 90 there are 5(8) = 40 pieces of uttecotch and 2(8) = 16 jelly Beane inthe jr Method 2: Parts ofthe whole. The butterscotch and te jelly beans are the only candles in the ar, and they have ratio 5: 2. Therefore, we 5 2 2 the total candy and that ely beans make up = 5 = thareae 56 pices otal thre are?» 56 = 40 pieces of uterscotch and 2-56 = 16 ely beans know that butterscotch makes up ofthe total candy. Thus, since 542 Next, we want the final ratio of butterscotch telly beans to be 2: 1. There are 40 pleoes of buterscotch and that wont change after we add jelly beans, so we want the new total amount jof jelly beans to be such that 40; j = 2: 1. You can probably see right away that we must have j = 20, butif nat, we can always compute it by multiplying both parts ofthe 2 I ratio by 20 60 thatthe first part ofthe ratio ‘equals 40, ke this:20) (1-20) = 40: 20, “This must equal 40: jj so wemust have j = 20. ‘We conclude that want the jar to have 20 ely beans. ft starts wth 16 jelly beans, sol need to add 20 ~ 16 = 4 lly beans tothe jar 2 Exercises ‘Simplify the folowing ratios: cba ide Solution Your Submission: Solution ‘Solution: The greatest common factor af 20 and Bis 4. Dividing out this factor, we vs You have hs Hide Solution Your Submission: Solution Solution: Since 6" = (2+3}"= 2*- 3" and 8" = (2+ 4)" = ceommon factor we have 4 the greatest common factor of 6 and 8 is 2°. Divicing by this 2 ee ehate Hide Solution Your Submission: Solution ‘Solution: Multiplying both fractions by 10 tums each into an integer: Sy 3 34 =10.3.40 5107 5 ( 10:50 Preview: Solution a type any ations notes Hide Solution Your Submission: Solution Solution Dividing both 100 and $00 by 100 gives 100 500 = 20%, 500 Seton Diving both 109 an 500 by 100 gives 100 500 = Ts @ 2.38 Hide SoationYour suomission: solution 1 20 Solution: Fst we wie both mixed numbers es fractions, which gves us 2 > Then multiplying both by 8 tums both Into imegers: 9 a © 672:0 Hide Soation Your Submission: Solution ‘Solution: Dividing both parts ofthe ratio by 672 gives a simplified rato of[1= 0} card bled There ate 10 boys ina class of 25 students, Whats the ratio of gis to boys? Hide Solation Your Submission: Solution Solution: There are 25 — 10 = 15 gil, so the ratio of giis to boys is 15: 10, We can vide by 5 to simplify the ratio as 15: 10=[B:3| TAVes Se aed Gear A makes 2 revolutions for every 5 revolutions gear 13 makes. If gear A makes 36 revolutions in 1 minute, then how many revolutions does gear I? make in 1 minute? Hide Soation Your Submission: Solution Solution: The ratio ofthe numberof revolutions A makes tothe number of revolutions 17 mekes ie 2 : 5. We multiply both parts of the ratio by 18 because A makes 36 revolutions: 2:5 =2-18:5-18 =36; 00 So, Bmakes [SO] revolutions when A makes 96 revolutions. We also could have used the 2: Sratio to note that 2 makes = as many evolutions as A. So, when A makes 36 revolutions, 3 makes 2 36-= 7 The ratio of gis to boys partiipating in inteamural volleyball at Ashland Middle Schools 7 to 4. There are 42 gi in the program. What ithe total numberof particinants? Preview: Solution Hide Solution Your Submission: Solution‘Solution: Since the ratio of gil to bays ie 7: 4, the ratio of gs to the total umber of studentsia 7: (7 4-4} = 7: 11 Multiplying both parts of this ratio by 6 to produce 42 gids gives us 7: 11 = 6-7: 6-11 = 42: 66,00 there are[65 total students, n We also could have used the 7 : 11 rata to eee that the total numberof students is " times the numberof give. Therefore, there u ered total students Two numbers are inthe ratio: 8, Ther sum fe 44, What isthe greater ofthe two numbers? ay type any eddttona nates Hide Solution Solution: Since the ratio of the smaller number tothe larger number is 3: 8 the numbers are Sin and 8a for some value of n. Therefore, we must have 3 + 8r = 44, so L111 = 44, Dividing by 11 gives = 4, so the larger rumberie 8-4 = [32] {An Binet-long submarine sandwich Is eut into two pieces whose lengths aren th tio F710 5, How ong is the shorter ple ye any addon Hide Solution Your Submission: Solution Solution: Since the ratio of the longer piece tothe shorter plece ls 7 : 5 the shorter piece | = FoF the whole sandwich Since the sandwich is 8 inches long the length of he shorter pece is 5-8 =[ 3 inches] = [3% inches cabs ead A father lft 280 acres of land to be divide among his sons Al and Bab inthe ratio 4:3, respectively. How many acres should Al ry ach een Jou have bg Hide Solution 280 = [160 sare “Two postive numbers are Inthe ratio of: 9. Ther fference is 30, it i the sum ofthe two numbers? ye ay adton Hide Solution You Submission: Solution Sol 9n — An = 30, s0.5n = 80, Dividing by § gives n = 620 the sum ofthe numbers isn +n = 130 = 13(6) ton: Sinoe the umbe fare in the ratio 4:9, the numbers are dnt and Qn for some value of n. Therefore, we haveTAICR acd The ratio of teachers to students in a partoulr schoo! is 1 to 17. The ratio of female students tothe total number of students is 4 to 9 If there are 296 female st ers ave there? Preview: Solution Hide Solution Your Submission: Solution Solution: First, we find the total umber of stucents, Since the ratio of female students to total number of studems is 4: 9, the total ° 9 number of students is {the numberof female students. Therefore, the total number of students Is 7» 396. Noticing that 396 = 400 — 4 allows us to compute this very quickly: 9 9 9 9 5 898 = 7400 — 4) = 5 -400— 5-4 = 900— 9 = 01. teachers There re 801 students, so mere are 91 n rAR CIS ad The ratio of losses to wins for Kyle's team is 3 to 2. the team had played the same numberof games, but had won twice as many ofits games, then what would the ratio of losses to wins have been? Hint Use a variable to write expressions for how many games Kyle's team won and lost. Preview: Solution Hide Solution Your Submission: Solution Solution: The ratio of losses to wins for Kyles team is 3 : 2, which means thatthe team won 2 out of every 5 games. If t had won twice 2s many games, but played the same number of games, then It would have won 4 out of every 5 games. This leaves 1 out of every 5 games to bea loss, So the ratio of losses o wins Is TAA Qe ‘The ratio of pennies to dimes in a Jars 2:5 and there area total of 245 pennies and dimes i the jr. How many pennies should be added to make the aloof pennies to dimes be 3 : 7? Hide Solution u may type any edltlonal nates you hev. Your Submission: Solution 3 the pennies nel dimes Inthe ja. There ore 245, Solution: Since the rato of pennies to dimes Ie 2 : 5, the pennies are == — = ites * 2487 pennies and dimes total sa there are = «245 = 70 pennies and 245 ~ 70 = 175 dimes. We wich to add pennies until the ratio of : a of the numberof dimes. Well stil have 175 dimes after adaing pennies to dimes is 2: 7, st which point the number of pennies is = 3 Peteyse es poet Tt pes dn aL. Tae ret eed 78 = Es ped7.2 Multi-way Ratios ‘Arata ts a handy gadget for comparing two quantities But is also useful fr comparing more than two quantities Far example, suppose 8 pet store has 4 dogs, Seats, and 11 golafish. We would say thatthe numbers of dogs, cats, end goldfish are inthe ratio of 4:5 : 11 Since 4 §, and 11 have no common factors, e cannot simplify this alo ary further Ifa diferent (more exotic) pet storehad 9 geckos, 12 Iguanas, and 21 snakes, then we would say thet the numbers of geckos, Iguanas, and snakes are n the rato of 9 : 12 : 21. But now, since 3s a common factor of 8, 12, and 21, we can divide each term of the ratlo by 3, to say thatthe numbers of geckos, Iguanas, and snakes are in the ratio of 9 osm = 3 Just 2s with two-way ratio, a multiway ratio only gives you Information about the relative quantities of the items-It doesnt tel you anything out the total numberof tems, For example, you know that third pet store has hamsters. guinea pigs, and rabbits nthe aia 12:5, all you know is that for every hamster, there are 2 guinea pigs and 5 rabbits. There might be 1 hamster, 2 guinea pigs, and 5 rabbts, or there might be & hamsters, 12 gulnes pigs, and 30 rabbis; more generally, there aren hamsters, 2n guinea pigs, and 5 rabbas for some number, Although the muiiple pats of @ multtway ratio may at fist look confusing, they relly not that much diferent from a two-way rato, as, ‘we wil seen the fallowing problems. Problems. Problem 7.6 © Jump t Solution Simplify the following ratios: (@ 5:15:10 @) 6:10:29 (@ 8:92:4:8 bet 33 @ sheat Problem 7.7 © Jompto Solution ‘A bowing tournament pays aut prizes tothe top 3 players inthe ratio 5 2:1 the total prize money fe 1.000, hen how much does ‘the first-place winner receive? Problem 7.8 © lump to Solution 1 ‘Sam wants to bake @ cake that requies butter, lu, sugar, and milk inthe ratio 1: 6: 2:1. Sam has cup of sugar How much of the cther ingredients does he need? Problem 7.9 ‘Source: MATHCOUNTS. © Jump to Solution Jamal needs thee gallons of @ mix that Is two parts blue pain, three parts white pant and one part red palnt. How many gallons of red paint will he need? Problem 7.10 {© Jump to Solution ' have blots, bleets, and bis In a bag, The ratio of the numberof bots tothe numberof beets Is 3 : 4. Te ratio ofthe numberof bleets te the numberof bits is 5 : 6, What isthe ratio af the number of ots tothe number of bts? Just as witha two-part ratio to simplify a multiway ratio means to write the ratio using integers with no common factor larger than 1, So, the ratio 62428 Is not simplified because each partis multiple of 2, wile the ratio q 2 Isnt simplified because the parts ae not integers. Problem 7.6 @ee ‘Solution fr Problem 78 (@)_ The greatest common factor of 5, 15, and 10 85, s0 we divide each part ofthe ratio by 5 (b) itis litle more dificult to see what the greatest common factor of 6,10, end 9s, We can get a litle more Insight by factoring each umber: 6=2 3, 10=2-5,9=3-3 Now we can see that there is no factor greater than 1 that is common to all thee numbers, co the ratio 6: 10: 9 is already simple. (6) Wenotice that 4 divides alte terms ofthe ratio, co we have (2) We handle fractions a Ile bit cifferenly: now we want to find @ number we can multiply each term by so that each becomes @ tol integer. This is usualy the least common denominator of the factions. nthe ratio: 5: the common denominators ©, 50 we rmattiphy 1 PEE G IG) 004 (@) Mixed numbers ony look more complicated~theyre relly just the same as fractions. We normally fine them easter to work with we ‘convert them to fractions: S348 Then, asin part (6) above, we multiply by the least common denominatar, which in this example is 8-4-5 = 60, to convert the ratio tats 1 17 26 _ (10 " 26 sers7(3'%):(7-2):(F-9) = 200: 255 : 312. Just as with a twoway ratio, we can tink ofa mult-ay ratio in terms of parts ofthe whole’ asin the next problem. Peas oi eee en ee ee ee ee ee ‘Solution for Problem 77: The 5 : 2 : 1 rato means that for every $5 the winner gets, the secone-place player gets $2 and the tir-place player gets $1. In other words, for every $5 thatthe winner gets, a total of $5 + $2 +8 $2 Tete. wer gets Pt eto ny iu Seep ory 81, 0rd ier ge Softee es $1,000 ~ $628 = In Problem 7.7, we saw that ratio of 5 2 : I edo the frst quantily being => = ofthe whole. nthe same way the sscond eae eae 8 7 ace winner gets 2 — = 2 _ + or he prize money, andthe tit place winner gets <*> — = ot the prize mone a eee BA - ee 2 res 8 ie * ‘Concept: Ina multipart ratio, you can often tink of each term as a ‘of the whole That i,you aoa a me terms i tne rano to get ne wnove, ana tnen eacn inavioust tem ee ‘makes up part ofthat whole WARNING Again, remember that ratio Is @ relative concept. The ratio only tells you the fraction of o ‘the whole that each part represents-It doesrit tll you anything about how much the total or each partis, conga Pen77 toate es atte ete wer St een Th drs i ct rich se wt gel ely aot sel pony a ween cone te ea money he Ci eee eee! Soe Re rnd ‘Solution for Prablem 7.8 Well present tree different methods for solving this problem, q ‘Method 1: Conver the ratio to match the given quanty. Te ge ratio of ute to four to sugar to mikis 1:6 2 1, But we onlyhave 5 1 ‘up of sugar, $0 we convert the ratio 30 that appears n the “sugar” position, Since the given ratio has @ 2 in that position, we need to “divide each term ofthe ratio by 4: ‘ 16.924, 91,1 liver repi ggg eg? gt 1 1 3 Now we ean just red fom the ratio the quentties ofthe other ingrecens that corespand to > cup of suger cup of butter, 5 cup of 1 flout and cup of mi ‘Method 2: Use separate twopart ratios. We can break up the muk-way ratio Into several separate two-part ratios, where each rato. ‘compares some ingredient to sugar. Far example the rato of butter to sugar fs 1: 2-this is just a ratio consisting ofthe butter and sugar ‘terms from the exiginal par ratio, Concept: We can remove terms from a mult-way ratio to get a simple rato that only compares & ome ofthe quantities from the original aio. 1 Since the butter and sugar are nao 1 2 we know that thee is half as much butter as sugar. We have cup of suger, 0 te amount of 1 14 butteris + 3 2 cup. 2 1 Silay, the rato of flour to sugar is: 2 = 3: 1, so there is thee times a much flour as sugar. Ths since there is 5 cup of sugar 13 tat there ae: + = 3 cups of our. Finally the ato of sugar to mile ia2: 1, sothereis + - 2 = cup of milk aan a - aaa Method 3: Compute the total. We can use the “parts of the whole” way of thinking with our part ratio. The sugar Is 2 201 1 115 ea = B= Pofine emtve ingredients, and we have 5 cup of suger Thus the total quantty of Ingredient Is 2 + TEE TEET ~ To ~ go the ent Ingress and we have 5 cup of sugar. Thus the total quant of ngrederts Is 5 + 5 = 5 cups. News it fie i epuwime Geared sar 2 0s oh 15 5 1 638 a5 8 008 of buter Newt the flour is — = ofthe whole eo there are 1 cups of four. inal the mik is 7 of w'2-% " 0 3 tavedlemstnnn om Lepetidtin w 2 = Concept: None of the techniques from Problem 7.8 are “right” or “wrong” Ratios are best Sepoaches wits Hele king Rats san be eres many ere aye ead you should use the method that you feel most comfortable with, arte method ‘that appears to work best forthe particular prablem that youre working on pee Breck eed Jamal needs three gallons of a mbx that ls two parts blue pant, three parts white paint and one pat red paint. How many gallons of red peter: Saluion for Problem 7 9 Since we ate aiven the total amount and we want ta fine the amount af ane of the nat. the Tarts af the wile”method wil probably work best. The given ratio Is 2:8: 1 of blue white ‘ted, and the total © 3 gallons. So the red paint Is 1 of me whole and thus Jamal is = Zot me whole and thus Jamal neds & 1 Halon of red pant. Problem 7.10 oy oe 18 bag, The ratio of the number of bots to the number of beets Is 3 : 4. The ratio ofthe numberof beets Pein ea ie cen tae cee tee ‘Solution fr Problem 7.10: Leve make @ tle chat ofthe data were given: blots beets bleets lite Bi4 3:6 We'd be able to compare bits and blots ifthe number of bleets inthe above two rao were equal. So let's make them equall We can do ‘this by muliplying the parts ofthe fst ratio by 5 andthe parts ofthe secand ratio by 4 blots: blest blest lite 15: 20 a 2h [Aha-now we can wite't as a 3way ato of blots to beets to bits. blots : blests + blits wo 20: wh The problem asked us for the relationship between blots and bits, and we ean read this information from aur Sway ratio. We see that blots and bits aren the rato 15 : 24, which canbe simplified as. blots: Dil ‘Jog winose leat is 60 Inches is cut into three pleoes inthe ratio 1:3 25. W piece? it Is the number of inches in the length ofthe s yoe any adltional Hide Solution Your Subs lution: The shortest pace is tics] = [6 nce Three numbers haverrtio 1:2 3, and theirsumn ls 49, What isthe greatest ofthese three numbers? 3 any edition nates you Hide Solution Your Submission: SolutionSolution: The greatest of the mumbers Is eee eee i largest number is + 48 = largest numbers 5-4 ree Se acd Purple palnt is made wth a 16 : 3:1 ratio of How much white paints needed in orderto make one gallon of purple paint? ay ype any addtional notes you have he Hide Solution Your Submission: Solution 16 1 4 Solution: The white paint is ——~"—— = = of the mixture, so = 1 is needed to make 1 gallon of purple paint. +341 5 5 * Eee ree = Three fiends, Aki, Bruno, and Carmela, pooled ther money to start a lemonade stand. Akira contributed $25, Bruno contributed $20, and Carmela contibuted $35. After @ month their lemonade stand had eared $2,000, and they want ta distribute this money in the ‘same ratio as the money that was invested, How many dollars wil runo recehve? Hide Soation You Submission: Solution ‘Solution: The rato of Akira, rund, and Carmela investments is 25 : 20 : 4 4 Byas7 ~ 16 dollars 5. Dividing all parts of thi ratio by gives 5: Therefore, Bruna invested L 1 = 1 of the money, so he should receive + of the earnings. Therefore, Bruno will a a : 1 reese | ‘The top four winners in a golf tournament share the prize money inthe ratio 1 fthe top prize winner recelves $45,000, then how much prize money is awardedin total? ye any adltional Hide Solution Your Submission: Solution this ratio by 5000 to make the top winner equal to 48000 gives 45000 : 85000, co the total prize poo! i $85,000] ‘Alex owns three times as many brown shoes as red shoes, twice as many black shoes as brown shoes, and four times as many white shoes as red shoes. What Is the ratio ofthe numberof white shoes to the numiber of black shoes he owns? ofthe total, 0 the ratlo of the top winner tothe total is 9: 17. Multiplying Hide Solution‘Solution: From the information inthe problem, we have brown : red black : brown white : zed Bid aed aid Conveniently, reds numbers the samelin the first and third ratios. So, we can combine these two ratios and write brown : red : white black : brown Bi dia eo ‘To combine these two ratios, we need the number corresponding to brown shoes to be the same in both, So, we multiply both parts ofthe second ratio by 2: brown : red : white black : brown Bi 1:4 6:5 Now, we ean combine the ratios black : brown : red: white 6: 3 Tid So, we see thatthe ratio of white shoes to black shoes ia 4: 6 which equal wee pee acd 1 Trae stings have a gift of $100 to elt inthe ratio of 5: +. What she grees numberof dollars that any ofthe ings wl Hint Can you write the ratio in an easiero-use form? Preview: Solution " Hide Solution Your Submission: Solution Solution In order to make the ratio easier to work with, we start by simplifying ft. The least common denominator ofthe fractions in the ratio I 12, 80 we multiply al parts ofthe ratio by 12. That gives 12 6 © ofthe money. Since there is $169 to divide, the one who Therefore, thesbing who gets the most mone receives =O =7.3 Proportions Whenever we have two ratios that are equal, we have a propertion. The most common usage of proportion is when we have two changing ‘quantties that ae related n such a way that ther ratio doesnit change. For example, suppose that Maro's secret recipe for chocolate milk uses 8 ounces of mik and 2 ounces of chocolate syrup, and produces a ‘D-ounce glass of chocolate mik. The ratio of mik to chocolate syrup is 8: 2 or 4:1. IF Mario wants to make a big pitcher of chocolate ‘milk for 6 people, hen he wil ned 6 -8 = 48 ounces of milk and 6 -2 = 12 ounces of chocolate sytup, go the rato of milk to chacolate syrup i 48 12, which stil 4: 1. No matter what quantity of chocolate mic that we want, the rtia of milk to chocolate eyrup will aways: be: 1, We say thatthe mik and chocolate syrup are proportional rin proportion Problems Problem 7.11 {© Jump to Solution Charlotte is planning a vacation to Europe. The exchange rate is 1 dollar equals 0.Geuros, or $1 = £0.60. If Charlotte wants to for her rip, then how many dollars does she need to convert? 300 Problem 7.12 ‘Souree: MATHCOUNTS. © Jumpta Solution 1 ‘recipe calls for 2 cups of flour and 4 eggs. If only 3 eggs are used, then how many cups of flour should be used? | Problem 7.13, (© Jump t Solution Sadie ie 3 fect tall and at & pm. in Sunnytown, Sacie caste an foot shadaw Nick is © feet tall. How long is bis shadow at 6 p.m. in Sunnytown? [Problem 7.14 {© Jump t Solution 1 é , My map of upstate New York has the scale + inch ~ 5 miles Buffalo and Albany ere 12 inches apart on my map then how far apart the cities? Problem 7.15. ‘© Jump to Solution Sylvia isan architect designing @ new building. The building will be 30 feet tal, and the windows will each be & feet high. Sylvia draws blueprints forthe building on which the building f 8 inches tall How tall are the windows on Sivas blueprints? Problem 7.16 ‘© Jump to Solution by wallet-size photo of my pet cat Snockums Is 3 em wie and Sem tal. If] want larger photo to put on my wall and I want the area of the photo to be 135 square centimeters, then how many centimeters wide should the larger photo be? Problem 7.41 gy Charlotte ie planning vacation to Europe. The exchan et eee te ee Soliton for Problem 7.11: Since $1 equal €0.G0 the ratio between an equal amount of dollars and euros Is 1 : 0.6, which simplifies to 5: 3 Welet be the numberof dollars that equals €300, whichis what we want to find. Then as a proportion we have 5:8=2: 300, ich gives 3 = ‘We can sole forza. = 300. > = 500, Hence, Charlotte must convert $500 in order to receive 300. eran Erne Wed i ee eee ee ee ert Se Solution for Problem 7.12: There are two primary methods we ean use, 1 Method 1: Set up a proportion by equating ratios. Because te expe cals for 25 cups of flour and 4 eggs, we know tat the rato of Nout to L eg should always equal 25 «4. f we only have 3 eggs, then we need an emount of Nout so that the rato of our to eggs stil equals 14. 1s means wat weave cups 01 Hour 10 yo wl OU segs, we US Have We solve this by wing the equation with fractions: "Multislying both sides of this equation by 12 gives 3 1 5 4 Therefore we need = Loupe of fou. 3 Mettod 2 Scale the quanties: Because we only have @ eggs andthe recipe calls for 4 epg we ae only using ofthe recipe amount cups. In the nest problem, we use the geometic fact that at any glven time, the length of an object is proportional to the length ofits shadow, (This is based on the geometric concept of similarity, which you wil learn when you take @ geometry course) pres ea So Lo Ce Le eee! Solution for Problem 7.12: The information about Sadie tells us that the length of an object (or person) and the length offs shadow are 3 5 proporonl nthe rato 38 If Nick’ shadow has length then the ratio 5: must equal the ratio 3: 8. Therefore, > — 3, and 0 oat 1 imutilyng both sides by Bx gues 40 — Sr, 20.4 — 5" — 13.5 Thus Nick’ shadow 6185 fetlong. = a GEnacs er Peony 5 Dee acs Pore Solution for Problem 7.74: A map's scale Is another example of proportion-it tells us thatthe ratio ofthe distance on the map to the Cltance inthe real world i constant. For my map, this ratia is l > inch: 5 miles 4 "Multinlying by 4 simplifies this rtioto 1 inch : 20 miles, so that 1 inch onthe map corresponds to 20 mile neal if, Thus the 13 inches. lon the map between Buffalo and Albany means thatthe eles ae 13-20 = 260 miles apart. = Eernray Pete ee eo eee ee ee eet ee eee eed Soliton for Problem 7.15: We stant with te ratio 80 foot: B feet = 90: B= 15 24 Inulin height : window height “Thus, on the blueprints, we must have the same ratio betwcen the heights ofthe bulling and the window. Sof the window height on the blueprints is 2 Inches, then we have 6 :4=8:2, ws 2 2 so that [P= =, Multihing both sides ofthis equation by a gies I ie Thetfore, the windows are 25— Inches tall on the blueprints, = Enna Pi eee esary then how man the ph ger photo be? Solution for Problem 7.16: The assumption inthis problem is that no matter what the size, the picture will ays have the same shape, ‘More precisely this means tht the ratio of width to height will remain constant. Since the wallet-size photo fas with 3 cm and height 5 cm, the ratio of width to height wll lays be 3 : 5. But were given neither the width nor the height ofthe larger photo: we're only gven the area, So how do we set up 2 proportion? ‘The proportion tells us that the width is Sar em and the height fs Six cm for some number 2: This means that the area is (82) - Giz) = 154? square centimeters. If want a larger photo with area 138 square centimeters, then we must have 15x” = 135, or 2° = 135/15 = 9. Therefore, x = 3. So my larger photo willbe 3x = 3(3) = 9 om wide (and 5x = 6(3) = 15emhigh).c Exercises ve peer acs ‘Aream of paper containing 500 sheets Is Sem tick. How many sheets of his type of paper would there ben a stack 7.5 om high? Preview: Solution ide Solution Your Submission: Solution 500 Solution: Let there be 2 sheets in the 7.5 em stack, so we have 500 Writing this using factions, we have 80 = = 100, Muitipiing both sides by 75 gives ’An American traveling in Japan wishes to exchange American money (dollars, symbol $) for Japanese money (yen, symbol ¥), Ifthe exchange rate is $1 — Y80, then how many dollars wil the traveler need to purchase ¥ 10,004 ide Soation Your Submission: Solution Solution: Let be the numberof delas the traveler needs. Since the rato of dollars to yen ls 1: 80, we have 1 : 80 = x : 10000, r 10000 1000 Malipjng both sides by 10000 gives = “TO = 1 Wiring this using fractions gives = = = = 8 39 — oOo yk pen acd ‘Alexia designed a logo? inches wide and 1.5 inches tall tobe uced on her schoof website. The school wants the logo to appear on the website as 8 inches wide, How tall wl the logo be onthe website ft is enlarged proportionally Youmay type any ad 3 Hide solutionSuk 1 WR UF WEDS YOYU EN nye as Ea LE leon Alii ign or 4-.5 ‘A bank has two flagpoles next to each other. Ifthe taller 30foot ole (fying the US. flag) casts @ shadow of 20 feet. and the shorter Aagpole (hing the state flag) casts a shadow of 15 feet, then how tall s the shorter flagpole? yi Sure Hea Ure went yD HHUNS aN Ue ARES ide Soation Your Submission: Solution Soliton: Considering the taller object, we have Object height : Shadow height So, we have Short pole height : 15 feet = 3 ‘Multlying both parts ofthe ratio onthe right by 7.5, we have Short pole height 15 fect = 22.5 «15 So, the height ofthe short pole 16 have reasoned as follows, The shacow ofthe shorter ole ie" ofthe shadow of he taller pole So, the height of the We might a 15 lerpole. Therefore, the height ofthe shorter poleis 2» (0 eet) 6 poleis Sr ofthe height of thet sents 50 miles, then what is the number of miles represented by 2 tr Hide Solution Your Submission: Solution Solution: We have Map : Actual distance = > inch : 50 mils. ‘Multiplying by 4 makes this ratio 1 inch : 200 miles. So, each inch on the map represents 200 miles Therefore, 2 Inches represents 200 (3) =200 (2 ) = [ini] PEA acd [Acrafteperson makes a scale drawing of a 100 meter % 30 meter bullting, where 1 centimeter represents 2.8 meters. How many ‘centimeters are in the smaller mension ofthe drawing ofthe bullng? de solution Your Submission: Solution Solution: Let 2.om be the smaller cimension inthe drawing. Since we must have Drawing : Building = Lem : 2.5 mMultphying by 20. gives we have em:30m—1em:2.m, Wrting this as fractions, we have 300, = 12 So the emalier dimension ofthe drawing le pen acd ne Tele fiends met fr dinner at Oscar Overstufed Oyster House, and each ordered one meal. The portions were eo large, there was ‘enough food for 18 people. If they share then how many meals should they have ordered to have ust enough food forthe 12 of them? de Solution Your Submission: Solution Solution: The 12 meals they ordered were enough for 18 people, so we have the ratio Meals needed + People = 12: 18 = 2 a So,weneed 5 as many meals as we have people, Therefore, to feed 12 peopl, we need 5 «12 —[B] meas Pear a peer acd For every 3° rise in temperature, the volume of a certain gas expands by 4 cubic centimeters. Ifthe volume of the gas is 24 cubic ‘centimeters when the temperature 1632", then what was the volume ofthe gas when the temperature was 20°? Preview: Solution ide soon Your Submission: Solution Solution: As the temperature rose from 20° to 82°, t rose by 3” exactly 4 times. Each time the temperature rises 3 the gac increases by 4 cubic centimeters. So, as the temperature rase 4 times by 3, the volume expanded by 4 centimeters. After this expansion, the volume of the gae is 24 cubic centimeters, go the volume of the gas before the expansion was. =the volume of 4= 16 cubic7.4 Conversions “There isa special ype of ratio that is usefl for converting between different units of measurement Let's illustrate how this works with a simple example ‘We know that there are 12 Inches in afoot and there are 3 feet ina yard. Suppose you want fo use this information to compute the number ‘of Inches in a yard, Of course, you can probably do this problem in your head, or you may even have the answer memorized, But lets carefully work through to methods that we ean use to solve the problem, These methods wil help us work through harder conversion problems, where the answer isnot so obvious ‘Method I: Setup ratios. Well use a method similar to Problem 7.10 (the problem with te bits and bleets and blote). We can write ratios to ‘express the relationships between the units: inches + feet feet: yards wid Bd ‘Wed keto combine this into @ sway ratio relating all three units. To do that, we need the feet” amount in both 2way ralos to match, The ‘easiest way to do this isto multiply both parts ofthe ist ato by 3: inches : feet feet + yards 3: 3 Bd Now we can wite tas 3 3.iay ratio inches : fect: yay = 36:31 Removing the “feet” ges us a ralo of inches: yards = 36 : 1, so there are 96 inches ina yar. ‘Method 2: Use conversion factors. we write the ratios from Method 7 as fractions, then we have what are called conversion factors. To help us keep track of what's going on, well write the units as part ofthe fraction, So we'd write 12 inches 8 fet. Tieet T yards (For consistency, we usually ste al unit in plural, s0 we write the werd ooking "1 feet” insted of "I foot") Multaying the conversion factors together vill cancel he feet” an leave us with a conversion factor relating inches to yards: 2 inches ices _ 36 inches Tier" Tyan Tyarde So there are 36 inches na yard Compare the to methods used above. They'e relly the same tng! The conversion factors from Method 2 are just a convenient way for us to keep track ofthe ratios fram Method! “There are wo key Ideas to keepin mind when using conversion factors. Fist Concept: Think of conversion factors a fractions that ae equal to 1 & For example, we know that there are 12 inches ina foot, so we think of 12 inches feet. In other words, the quantities “12 inches” and "l feet” are equal quantities, sot makes sense to wrte them equal to each other in an ‘equation. Going one step further, this then makes the fraction 12 inebes Tieet “The right way to think about this is 2s a fraction with equal numerator and denominator, so of course is equal to 1. Also, we can just as easily wre ts reciprocal too: feet Wines (Our second key idea about conversion factors I: Concept: We multiply conversion factors together to cancel units, & ‘We see this concept if we revisit our earlier computation: 12 inches 3 fee: Tieet "Tanke All we're doing Is multiplying two fractions on te left thet are each equal to 1, 50 naturally (since 1 1 = 1) the quantity on the rghit side ‘ofthe equation is also equal 1o 1. But we also notice thatthe "eet" units eancel, You might See this more clearly I we putin some missing steps inthe above calculationJoos Toot toot _ (U2 wsches) yards (Tet) -( 7 yards) _ (12-3) fines fet) (1-1)(feet™ yards) 36 inches Tyards The beauty ofthis thatt helps us prevent mistakes. For example, if we incorrectly tried to combine conversion factors as: 12 inches L yards __ (inches - yards) Tieet " B fect I feet feet) ‘we see thatthe units dott cancel properh, 50 we probably made a mistake somewhere Conversion factors make it easy to convert units. For example, suppose we want to convert the length “6 yards" into inches. We know that 2 conversion factors just a fraction that equals 1, and multiplying “6 yards" by 1 doesnt change the length. Therefore, we have 36 inches 6 yates = (6 yards) «1 = (6 yards) But now the units nicely cancel and we can finish the compunation: 36 inches _ 6-36 inches 6 yards = (6, yands) «> = 216 inches Notice how the yards" units cancelled, and we are eft with ust the “inches” unts, as we want. Therefore, 6 yards ls equal to 216 Inches, crs Part Problems Problem 7.17 © Jump t Solution | How many yards equals 90 inches? Problem 7.18 (© Jump t Solution A tablespoon is haf ofa fluid ounce, a cup is 8 fluid ounces, and a gallon 's 16 cups. How many tablespoons ar in a gallon? Problem 7.19 © Jump to Solution Wl 10k 81,000 on his trp to Japan, where the exchange rete between dollars and yen ie $1 — 0. He spent ¥45,000 on his hotel room and ¥ 11,250 on meals and souvenirs. How much mney (in dolar) did he have remaining atthe end of his trip? Problem 720 Jump to Solation | The densty of waters approximately 8.3 pounds per gallon and there ere 4 quarts ina gallon. How much does 7 quarts of water welgh? Problem 7.21 ump to Solation an centimeters, Approximately how many square centimeters arena square inch? is approximately 2 Problem 7.17 ee ee Solution for Problem 7.17: We discovered above that 1 yard ls equal to 36 inches, To convert 90 inches into yards, we start withthe ‘quantity "90 inches" anc multiply bythe aporopriate conversion factors until we get something withthe units "yard In tis ease, i's easy ‘we ust need a conversion factor with inches inthe denominator (to cancel the inches in our ital quantity) and yards inthe numerator (so ‘well be left with yards), Our calculation isyards _ 90 s, 90 inches = 90 ines - FONE = FP yards = 2.5 yan Thus, 90 inches is the same as 2.5 yards. = Concept: The reason the calculation in Problem 7.17 works is thet the conversion factor is 1 yards a ‘equal 01. Thats, Because 1 yards ~ 36 inches the fraction =F" Is equal to 1. Therefore, multiplying 90 inches by this fraction isthe same as multiplying by 1 and hence the length does not change. rer ee eee eet ae nee ad Solution for Problem 7.18: We want to convert"! gallon” Into same number of tablespoons. But we dorit have @ single gallonsto- tablespoons conversion factor. Instead, we have to use the multiple comersion factors that we are given inthe problem statement. First, ‘we show @ step-by-step solution. ‘We first convert gallons to cups. We dorit really need a "conversion factor for this since we ean just read this data from the problem statement 1 gallon = 16 cups. Next we convert cups to ounces, using the cups-to-ounces conversion 1 gallon = 16 cups Bounces Teups (16-8) ounces = 16 cups = 128 ones, Finally, we convert ounces to tablespoons (abbreviated thsp): 1 gallon So there are 256 tablespoons in gallon. But conversion Factors ate nice in thet we can use mare than ane of them at the same time. In particular, we could do the gallanto- tablespoons conversion all at once: 16 cups Sonnces 1 they 1 gallon = 1 gallon» SE, SOE 2 Tallon Teape F ouness 16.8 [As long as we are sure that each conversion factor equals 1 (meaning that its aumerator andl denominator epresent the same quantity) ‘and thet the uns cancel properly, we can line up 2s many conversion factors as might be necessary to do @ complicated conversion. = We can use conversion factors whenever we wish to compare two diferent units-they don't necessarly have to be typleal “measurements lke length or volume. For example: RC To Pee renee ne Learnt i rt erg i i ac i in alba a Wp te so 1.000. ¥8 — on 000 eae, a0 "ii i a iy ei i ci cy A i oma a en a xa en avatlrrieel ucac tienen ty 90,000 — ¥¢45,000 — ¥11,250 38,750.‘When Wil got home, he converted his remaining yen back into dollars: $1 _ 33,750 493,700. 5 = SE — sa So Will had $276 dollars remaining after his tip ‘Method 2: Conver Just the spending, Rather than convert Wills entie bankroll to yen and then convert back we can Just figure out how much he spent in dolar, Hs total spending was ¥45,000 + ¥ 11,250 = ¥56,250, son dolar tis is ¥56,250- a oo “Thus Will spent the equivalent of $625 on his trp, end had $1,000 ~ $625 nes) Cra ‘The density of water Is approximately 83 pounds per gallon and there are 4 quarts ina gallon. How much does 7 quarts of water weigh? ‘Soltion for Problem 7.20: We want to comet from quarts to pounds. We can use two conversion factors: one for quarts to gallons, and ‘one for gallons to pounds: 1 gallons 8.3 pounds T quarts =7 quarta- ENO , BS pou 7-83 T= pounds a Tauarts 1 gallons = 14.525 pounds Ee = ‘Solution for Problem 7.27: Methad 1: Reason geometrical. A square inchs the area of a square that i 1 inch on each side. But this same ‘square is 25 centimeters on each side, soits areas (2.5)(2.5) = 6.25 square centimeters ‘Method 2: Use conversion factors. We wish to convert from square inches (written in) to square centimeters (wrlten em). So we need to ‘cancel the inches units fice ane be left with the centimeters units twice. Thus, we need to multiply by two conversion factors: 25m 2, Tin 2.5)(2.5) em? Exercises Suppose that one US dollar is worth C8125 in Ganadlan dolas. if want to buy a C315 book in Canada, then how many US dollars do = Hide Solution Your Submission: Solution Solution: need 1 US dollar for every 1.25 Canadian dollars, so my 15 Canaclan dollars convert to 15/1.25 = 12S dollars. We can also do this using a conversion factor csi5 = e815, Ussi2 Basketball center Steve Tootall ie 7 feet 2 inches in height. What is Steves height in inches? Preview: SolutionHide Solution Your Submission: Solution ‘Solution: Each foot ia 12 inches, eo 7 feet is 7 12 = 84 inches. Adding the 2 extra inches gives 84 + the atea, in square centimeters, ofa equate thats = feet long on each side? Hide Solution Your Submission: Solution 1 1 ‘Solution: First since there are 12 inches in foot, there are 12 «= = 6 inches in 5 foot. Thus the square ls 6 Inches per side We could frst convert the side length to centimeters, alving Gin =6in = (6-25) om =15 em, 6 in PS = 6-25) cm =1 Thus the ares ofthe aquareis 15 - 15 Alternatively, we could use the fact fom the text) that 1 in? = 6.25 em? and convert the 36 square inches of area to square centimeters: 6.25 emt Tint Ad ald Ther approximately 28.35 grams in an ounce, and 16 ounces in a weigh? Round your answer to nearest whole numberof grams. nd, How many grams does a quarter-pound hamburger Hide Solution You Submission: Solution ‘Solution: We ine up the conversion factors: 16 onnoes 28.85 grams au apa cee i 5 cups of flour. Unfortunately, Natalya hes lost all of her messuting 1 lespoon, = ounce in tablespoon, and 8 ounces ina cup. How many Natalyals secret reepe for ps nut butter cookies calls for 2 equipment except for 9 teaspoon. There are 3 teaspoons ina teaspoons of flour does Natal ed for her recip Preview: Solution Hide SolutionYour Submission: Solution ‘Solution: We ine up the conversion factors . go, Soe the 3 tsp 25 cups = 2.5 epee a 58:3 | = = 120 tm. Thus Natalya needs [120 ] teaspoons for her recipe. cA eis 1000 meters s 1 kilometer, and 100 hectares is one square kilometer. How many square meters are n 1 hectare? 1 V9 any addtional note ehere ide Solution You Submission: Solution ‘Solution: We have (note ha s shot for hectare): eee (ne eM TOO a (Tk cee ei (On the planet Qinbob, the unit of currency isthe Ploktr, the unt of weight is the stuun, and the unit of volume ie the piquat. The precious liquid vimiy is worth 400 Ploktar per stuun and haa a density of 20 stuun per piguat. f Aanie has 500 piquat of vrmuy then how many Pioktars Isher virwy worth? Hint Doi tthe weed words foo you this problem s|ust ike many others youve solved inthe section, Preview: Solution ide Solution Your Submiss(an: Solution ‘Solution: Despite the foreign words, we’can sll use eoTwersion factors! 20 stun 400 Ploktans Tpiquat "1 stun = (500 20-400) Ploktars = [000,000 Ploktars 500 piquat = 500 piquat7.5 Speed ‘You are probably already familar with the idea of speed, Speed is a measure of how fast something Is moving, For example, suppose a ca is traveling ata constant speed of 40 miles per hour. After 1 hour it has traveled 40 miles; afer 2 hours it has traveled 60 miles; efter 3 hours, thas traveled 120 miles, and soon. Notice that te rato of distance traveled (in mils) to time traveled (in ‘rours) is alvays constant: 40:1 =80:2= 12023, ln general ifthe car travels for hours, twill have traveled (kr miles, for arti of distance ta time of A0kr
— 68 miles in each hour. Therefore, ts speed 1s 68 miles per hour. ‘Again, we can think about units to realize that we should dive, Our answer isa epaed, 6 ts units should! be “miles per hour” Thus, ‘we divide the éstance (in miles) by the time (in hours) lstane speed = ime 238 ales ~ 3.5 hous = FE lls por hour 8 miles per hour, 420 (©) Hamotoreyele covers 80 mies ineech hour then twill cover 420 miles In > = 5.25 hous ‘As with the first two parts, thinking about units can help us see why we divide. Qur answer is time, so should be expressed in “pours Thus, we have to dhe distance (in miles) by speed (i miles per hours) o end up with an answer in terms of hours: dlistance _ 420 miles 420 time = = hours = 5:25 hours 30 speed 220 420 ales - OR — SP hours = 5.25 hour. WARNING! ‘There's nothing special about ‘miles’ and “hours” In Problem 7.22. Speed can be o ‘expressed in any distance unit per any time unit. So it might be given in meters per second, or fet per minute, o light years pe fortnight. Just be s units consistent! rethat you keep youra this rate, how many miles will the train travel 1 hour? a Solution fr Prablem 728: One minute 20 seconds is minutes, so the tas speeds 2 males peed = ~ 3 inate ‘There are 60 minutes in an hour, so the distance traveled in 60 minutes ia (6 inne -(2 222) distance 3 saa) = 40 ls Another way to solve Problem 7.23 Is to reason as follows: the number of "1 minute 30 second” intervals In @ 60-minute hour Is 10. The train covers @ mile m each ofthese 40 intervals. Therefore the raln covers 40 miles an hour Problem 7.24 Ere cd eee ee ete enor ner ne A eS Prep tetera nies an nyt Selation for Pelam 7.28 We can compute te waling stance an ranring tances Seperate then a them You walk or 48 Zhowrat arate of 10 mph othe nning stance (10 mph)(Z hour) = 5 wiles, So the total distance you over ie 3-+5 = Ses Now we can also compute the average speed for the journey. You covered 8 miles in joes, 0 your average speed was 8 males F hows Gees CY ‘Shelly drove the 50 miles from her home to her office at an average speed of 75 miles per hour. Coming home, she encountered heavy traffic and drove the same 50 miles at an average speed of 50 miles per hour. What was her av eats ed Solution for Problem 725: You might think Bogus Solution: Shelly drove hslf of her tip at 75 mph and the other half of her tp at SD mph. @ “Therefore, her average speed ie just the average af the speeds from the two halves ‘the tip, which Is (75 + 50)/2 1 125/2 = 625 mph. 5m ‘We cannot average speeds inthis way, a5 we wll see when we compute the amount of ime the tip takes, ‘The trp from the home tothe ofice was 80 miles at 75 mph, 20 8 takes 50 miles 2 Temmph ~ 3° 2 “The tip from the office tothe home was 50 miles at 50 mph, 60 It takes 1 hour Thus, the entire 100-mile tip takes 1 = hours of travel time, ‘and we can compute the average speed to beIo le 20 uni 17 bors “Therefore the average speed ofthe roundrp joumey Is 60 miles per hour = WARNING! Speeds do not usually “average” inthe way that you might expect them to, Po Sidenote: There Is relaonship between the speeds forthe two potions ofthe joumey In Problem 725. The average epee! fr the ene tp isthe Ranmonic mean of the FP speed for each nal of thet, given by Harmonie mean is another klnd of "average" that has many uses in advanced ‘mathematics ees) ora re ec pre econ eta heer a eres met Serra ‘Solution for Prablem 7.26: There are a couple of efferent ways we could approach ths. ‘Method T: Setup an equation Lett be the time of day n hours-t makes sense to make ths aur variable since this Is what we're tying to find inthe problem. We can also imagine the houses as being onthe number line, where Ben's house Is at O and Alsha's house is at 100. Ben starts cycling st 7 am, and he moves right (slong aur imaginary number line) at 15 miles per hour. So Nis positon at time t Is 15(¢ — 7) Alisha starts cycling at 8 arm, and she moves left at 10 miles per hour. So her postion at time tis 100 ~ 10(t ~ 8) (Note the ‘minus signin front af the 10, because she is moving ta the lft.) They meet when their positions are equa, so atthe ime that they meet we have 15(¢ — 7) = 100 10(¢- 8). Expanding this equation glves 15t — 105 = 100 — 10¢ +80, and simplifying gles 25¢ — 285, So ¢ = 285/25 = 11.4, Therefore, we ‘could say that they meet at 11.4 2m, but of course this fs not how we normally express the time of day. This is 04 - GO = 24 minutes past 17 am, so they meet at 11:24 a.m, ‘Method 2: Think about how the pzople are moving relative to each other We notice tet Inthe first hour (between 7 and 8 am), only Ben ‘moving, and he covers 1S miles. Therefore, a aim, the two people are 10K) ~ 15 = 85 mies apart After 8 am. since Ben and Alisha cover a combined 25 miles per hour between them they reduce the dstance between them at arate of 25 miles per hour. They will meet when ths distance Is reduced all the way 10. Since they stat with 85 miles between them at 8am. and ‘they reduce this ciatance at arate of 25 miles per hour ill take them 85/25 = 3.4 hours to reduce the distance between them al the ‘way to 0. So they will meet 3.4 hours after 8 am, whichis 11.4 arm. asin Method 1. Again, converting to minutes gives 0-4» 60 = 24 minutes past 11 am, so they meet at 11:24am.= Method 2 above uses the following important idea Concept: When there are two people or objects moving simutaneously, is often easiest to a keep track ofthe distance between the object, rather than trying to keep track of the ‘objects separate “Try this concept in the next problem. Problem 7.27 Ci oe kee ht th ete Cee oes einen rents eee Ae Lee Ce pre et ees rd ‘Solution for Problem 7.27 (a) We can think af Yaa! and Reo fia ae heina AON meters nar at the start eal thew have ton a nambined 40N meters nti theywill meet again, They reduce the distance between them at a rate of 5+ 3—8 meters per second, So they will mest afer 400/8 = 50 seconds, (©) When they run In the same direction, we think of the distance betiveen them as increasing. The rate at which the distance le increasing is the difference in their pres, which jz i — 3.= 2 meters per second, They meet when Yogi “laps” Boo-Boo, meaning when Yoot has increased his lead over Boc-Boo to 400 meters, or an enti lap, and thus Yoo! catches Boo‘Boo from behind, Since Yogi’ lead increases by 2 meters per seconde wll atch Boo Boo after 400/2 = 200 seconds Sidenote: _Consigering how two people or objects move relative to each other Is @ key concept In a solving many problems in physies. ‘Speed Problems Part 2 Exercises 3 (@) At50 miles per hour how far does a car in hours? i Preview: Solution Hide Solution Yow Submission: Solution Solution: A car thet travels 50 miles in hours goes w-( ) = [192 nies) (@) At 60miles per hour how long does it take @ cart travel 220 miles? Hide Solution Your Submission: So Solution: n order to cover 320 miles by traveling 60 miles pet hour the car needs to travel for 1 1 and + ofan houris + 60 qotlg sateen 8 10 minutes, a better way to write our answer is] (©) How fast does 1 have to travel t go 280 miles in 3 hours? Hide SolutionYour Submission: Solution ‘Solution: The epeed of the car isthe distance it travels dvded by the ime it wavele. Ifthe car must travel 280 miles ata constant 280 280 miles _ 280 miles _ 260 miles _ [> a hour ibour Tour rae ae (n a sunny July day, Mo starts at 10 am. In Calgary, at which time her car's odometer reads 27289 kilometers, At 4 p.m. she aries in ‘toon, at which time her eas odometer reads 27816 kilometers. (Mo didnot need to adjust her clock fora time zone crossing, use nthe summer the time in Calgary is the same as the time in Saskatoon.) What was he speed for the tip? Preview: Solution ype any adlton 7 Hide Solution Your Submission: Solution Solution: The car travels for 6 hours, and goes 27816 divided bythe time travels, 0 the cars speed is Pern per ied veled 80 mils in an hour and a half, then was stopped in traffic for 30 minutes, and then traveled 100 mi next two hours. What was the ars average speed for the Ahour rip? les during the ay type any additional notes you hav Hide Solution You Submission: Solution ‘Solution: Altogether the ear traveled 80.4 100 = 180 ilesin 4 hours, 20 te average speed is 180 miles _ 180 miles _[ miles Thowrs 4 our cae ee cd Peter had 2 12:00 noon appointment that was 60 miles fram his home. He dove from his home at an average rate of 40 miles per hour ‘and arrived 15 minutes late. At whatsime did Peter leave home forthe appointment? Hide Soation You Submission: Solution ‘Solution: Peter drove 60 miles at 40 miles per hour So, the amount oftime he drove was G0 miles _ 60 ale 2 ee 0 ours3_1 1 Se Peter arived 15 minutes late, he arved at 12°15 pm. Therefore since he drove 5 = 1; hours he eft his house 1°; hours before 12:18 pm, at[10: ABarm] cAsich Sled |Lusually walk from home to work. This morning | walked for 10 minutes until was halfway to work. then realized that | would be late if | et walking, Ian the rest ofthe way. lun twice as fast as I walk. How many minutes otal dit take me to get from home to work? Hide Soution Your Submission: Solution ‘Solution: Had | kept walking, the second half of my trp would have taken 10 more minutes. By doubling my speed for the second half of my trip, | halved the amaunt of ime i tock me fo finish, So, the second half of my trp took 8 minutes, fora total trip time of 10+5 To see why doubling my speed resulted in having my time, recall that distance time = spec So, Ifthe distance remains the same and we double “speed then time" must be halved. ca Atrain is traveling 7 mile every 75 seconds. Ifthe train continues at this rate, then how far wilit travel in two hours? Hide Solution Your Submission: Solution futon T: Seale the traveling time First, we convert the 75 secondsto minutes. We have 1 jnmte 75 soconts = (75 seconds) « mn - =F tunnts, So, the tain goes 1 mile every minutes. ‘Multiplying by 4, we see that the train goes 4 miles in § minutes. Multiplying by 12, the train goes 48 miles in 60 minutes, whichis ane hou So, we multiply by 2 to see thatthe tain goes to hours Solution 2: Multiply the train's speed by the time travels. The traln travels at @ speed of 1 mille per75 seconds. That I, ts speed Is 1 mil Fag Ritwvels at tis speed for 2 hours, 20 the distance raves is === «(2 hours), The problem now s thatthe Hours and the seconde dort cancel. We can use a couple of conversion factors to conver! the hours to seaonds: (2 hows) ile 50 msites 60 seconds 75 seconds Gk TV howe T minute L nile (2 ont (60 ane (6Dsecond) TB econ (ot) (ane) 1 nil (7200 sooo)Jason and Jeremy work tagether at a juggling ball factory. Jason lives 25 miles away from the factory and drives at 60 miles per hour Jeremy lives 35 miles away frorn the factory and drives at 70 miles per hour. If hey leave thelr houses at the same time, then who antves at the factory fst, and how longi it unt the other aces? Preview: Solution tes you have he Hide Solution Yur Submission: Solution Solution: We compute separately how long it takes each person to travel 10 the factory. Jason dives 60 miles per hour for 25 miles, sohis tp takes him 2% 5 = 2 hows = > hours 0 2 Jeremy dives 70 miles per hour for 35 miles, so is tnp takes him F hours 2 1 Jeremy. An hour ts 60 minutes, so Jeremy's = hour is 30 minutes, Jason takes 60 minutes 5 4 SES = 5-60 minutes = 25 minutes. So Jeremy arives 30 ~ 25 = [Bininuts] ser Jason ‘Arman érives from his home et 30 miles per hour to a shopping mall that Is 20 miles from his home, On the return tip, he encounters heavy trafic and averages 12 miles per hour. To the nearest mile per hour, what is his average speed for the roundtrip to and from mali? ide Solution Your Submission: Solution ‘Solution: The taal distance forthe round tripe 2-20 = 40 rrles. Next, we must compute howe long the round trip took. On the way tothe mall he goes 30 miles per hour for 20 miles, So, the amount of time he takes is 2 Combining these, the whole round tp takes 5 is 40 miles 40. miles _ 120 wiles hours 7/3 hour 7 hour ui oY Two doge run around a ereular track 200 feet long inthe same tection. One dog rune a a steady rate of 15 feet per second, the otherata steady rate of 12 feet per second. Suppose they start at the same point and time. What's the least numberof seconds that will elapse before they are again together? Hide Solution u may type any editions! Your Submission: Solution Solution: Since one dog i 3 feet per second faster than the othe, the faster dog's lead grows by 3 feet every second. The two dogs willbe back atthe same point when this lead grows to 200 feet, which is afl lap around the rack. Since the lead grows by 3 feet ‘every second, the lead reaches 300 feet in 300 eet _ 300 =p son er pee acd A train traveling at 30 miles per hour reaches a tunnel that is 9 tim ‘the tunnel, then how long isthe trainin feet? (1 mile equals 5280 feet) long as the train. If te tran takes 2 minutes to completely clear Hint: How many tai-lengts does the tain tavel to clear the tunnel? Hide Solution Your Submission: Solution ‘Solution: First, we determine how many train length the train travels during the 2 minutes takes to clea he tunnel, In order forthe train to clear the tunne, the front of the train must travel the entire 9 traintengihs ofthe tunnel, and then the tain must go 1 more trainength inorder forthe rest ofthe tein to leave the tunnel. So, the train travels 10 tralength to cleat the tunnel. This means that the tains pedis 10 train lengths per to minutes, or 0 traistongths 5 traimtongths 2 mines ‘on We know thatthe tran travels 30 miles per hou. So, if we convert our trainlengths per minute to tallengths per hour then we know how many trair-engths the train covers inorder to go 30 miles: Arain-tengths 60 minutes oq) trin-lents vainite rate Thou Thou 1 Therefore, in euch hour tha the tran goes 20 miles, covers 200 tainlenghs. This means thatthe train is = = = male long. Converting this to feet gives7.6 Other Rates Speed is just a special example ofa rate Whenever a quanthy changes bya certain amount in fixed unt of time, we havea rate, The des ‘of ates very lenible and can be used in aot of ifferent situations, Problems. | Problem 7.28 (© Jump to Solution | Jason can type at arate of 40 words per minute, How long wilt take him to type @ 2,000 word essay? Problem 7.29 © Jump to Soltion -Ahose ils swimming pool at arate of 0.5 gallons per second. I the pool's capacity is 9,000 gallons, then how many hours does itake forthe hose to completely fill an empty pool? Problem 7.30 lump io Solation | vate wants to give «45-minute speech, and she speaks 120 words per minute. Her waten notes contain S00 words per page. How many pages should she prepare? | Problem 7.31 (© Jump to Solution | ayvscarhas we ste cence 7h Chere oa el he Sane aah We How i mary revelutions wil hie tres make if Rav drives toa store that is rile avay? (Assume thatthe drive is completely straight, nd recall that 1 mil is 5.280 feet) | Problem 7.32 (© Jump to Solution | 15 woodchucks could chuck 50 cords of wood in 4 days, then how many cords of wood could 7 woodchucks chuckin 6 days? | Problem 7.33 {© Jump to Solution ‘Tom ean paint Mr. Thatcher’s fence in 6 hours, while Huck can paint Mr. Thatcher's fence in 5 hours. f they work together, then how long vl ttake them to paint the fence? Justa th epeed, the use ofthe word “per” likely 2 signal that we are working with arate, Algo, just as with speed, we can use the units {o.our advantage when solving problems involving rates. Heres basic example: ed By ‘Solution for Problem 7.28: Method 7: Dect reasoning. Jason needs to type 2000 werds, and for every 40 words that he will ype, he wil 2000 need minute, Therefore he needs = ‘Method 2: Canverson factor Jason's typing essentially converts irutes” ino “words” and vice versa, so we ean use 2 conversion factor ‘We needa arrange the deta so thatthe “words” unts cancel and we are lft with “minutes” uns: Liminutes — 2000 Fea = AIP minutes = 50 minutes, D minutes to type the entire 2000 words time = 2000 words So it takes Jason 50 minutes to type the essay. We can also easily check this answer: fhe types for 50 minutes, nd he types 40 words pet minute then he wl type a total of (50 minntes) ( ee ) = {50 - 40) words = 2000 words, inate as required. Concept: Use the units in your mathematical expressions to help you figure eut how to use the ag information inthe problem, ed meSR ony ‘Solution fr Prablem 7.29: First, we can compute how many seconds are necessary. This is usta basic conversion problem: 1 seconds 05 gallon seconds timo in seconds — 9000 gallons _ 000 =o = 18000 seconds ‘We can also convert seconds te hours using conversion factors. This gives Linimute 1 hour 18000 seconds = (18000 srconds) « FS « __ 18000 0-60 bors Thus twilltake 5 hours to filthe pool. = Concept: Unit conversions are Just another type of rate. {9 conversion factors, In particular, use a& units cancel n the way that you want. vith rates much lke we 1m in mathematical expressions so that the ee ‘Solution fr Problem 7.30: We have a couple of cifferent rates here: both the words per minute and the Words per page. We could do this a= «two-step problem, or use both rates et once, ‘Method 1: Count wards fist, than count pages. Ftst, we compute how many words should be inthe speech, Julie wants to talk for 45, ‘minutes ard shell use 120 words per minute, so she will speak (45)(120) = 5400 words. Then, we compute the numberof pages: 5400 ‘words at 500 words per page wil require 100/500 = 1.8 pages. ‘Method 2: Do all at once using conversion factors. We wee an expression that cancels the unis we dont want and leaves the unit we do want taumber of pages = (45 minutes). 220 onde 1 pase 45-120 500 = 10.8 pages. Troinutes * 500 words pages pee cr Raji’s car has tires that have circumference 75 Inches. (The circumference ofa te isthe distant eee en i Se ee ee that 1 mile i 5.280 feet) ‘Solution for Prablem 7.81: Weecan set this up as a product of conversion factors: number of revolutions i) 5280 foot 12 inches 1 revolutions a Tunlles Lfeet "75 inches 5280-12 =a = 211.2 revolutions revolutionsWork problems are another particular typeof rate problem Heres classic (slightly confusing) example Problem 7.32 ee See Lo en ‘Solution for Problem 7:32: What Information would be most useful? It would be helpful if we knew how much wood 1 woedchuck could ‘chuckin day. Fortunately itis not too cifieuk te igure tis out ¥ 5 woodehmicks could chuck 50 coeds of wood in d days, and since 1 woodchuck chucks = as much wood as 5 woodehueks, then we know that ie ct ae cata Ae 1 woorleiue could eluck 2.5 couds of wood in L day, then 1 woodchuck could chuck (6 2.5) cords of wood in 6 day. and thus T woodchucks could chuck (6+ 2.5 +7} cords of wood in 6 days Thus the answer ie 6-25 105 cords of wood. 2 Problem 7.33 Cy Se ee ne eee ree Pa eres 1 ‘Solution for Prablem 7.24: Our plan isto determine their work rates, and then add them. In particular, Tom can paint = ofa fence per hour, i yin and Huck can paint = ofa fence per hour So, together they can paint & + = = G7 ofa fence per hour. Therefore, the time to paint the wholefence is 4 fence 30 = F hows = 2 hours, Ti iences por hour 11 rr coralitle over2.7 hows. © “The key step in our solution to Problem 7:33 was considering how much work Tom and Huck each do per hour Concept: Wark problems can a ‘does per uni of time ved by considering the amount of work each worker ExercisesCasey has to build a 100-footlong fence, I complete the fence? takes her 15 minutes to build 1 Foot of the fenoe, than how many hours wil take her to Solution Hide Soation Your Submission: Solution Solution: Since it takes Casey 15 minutes to build each foot ofthe fence, ft takes her 15 - 100 = 150 minutes to bie all 100 fest of the fence. We can then convert the minutes to hours Lo ; 500 SST ee TD hous = We also could have used conversion factors todo the whole problem from the begining. Inge ee builds 1 foot pet 15 minutes, we have 1Gmimies _Lhowm _ 100-15 100 feet» mines, tHe 100 onre [SE hoa] Ticst OO minutes Phil can type. page of his new navel in 20 minutes. Ifhe writes for Shouts, then how many pages will he type? Preview: Solution ny adltonal Hide Solution Your Submission: Solution Solution: Phil produces 1 pa hours, he produces 3-8 = 20 minutes. Soin 3.20 = 60 minutes (which Is 1 hour), he produces 3 pages. Therefore, in 8 ‘Again, we can use conversion factors: 60 mimites Lp 8-60 Shou: Tour” 30 mintes ~~ 20 pages = 76: pee ics ‘A kangaroo chases a rabbit that strts 150 feet ahead of the kangaroo, For every 12foot leap of the kangaroo, the rabbit makes a 7-foot tne kangaroo have fo make to catch up tothe rabbit the two animals always leap atthe same time? leap. How many leap 3. any edltion notes you hav Hide Solution Your Submission: Solution = [BO times 0 Solution: The kangaroo gains § feet on every leap. The kangaroo must galn 150 feet total, so must jump catch the rabbit Maria buys computer dike ata price of 4 for $5 and sells them ata price of 9 for 85. How many computer diske must she sel in orderHide Solution Your Submission: Solution ‘Solution: We first determine how much money Maria makes per dick. Since she buys 4 dicks for $5, the per-dsk rate she pays i olan SGN Saucy oes eae ig Baolars _ 8 dears sels 3 disks for 85, c0 the perdi rate she receives le TOE = > EE ‘Therefore, her proft 1 disks i per disks dollars 5 dollars 2 disk isk Since thi isthe amount she makes for each disk the numberof dicks she needa to make $1008 ee ee disks = [240 disks {At the beginning ofa tip, the mileage odometer read 56.21 miles. The dver filed the gas tank with 6 gallons of gasoline. During the ‘rp. the driver file his tank again with 12 gallons of gasoline when the odometer read 56,560. At the end ofthe vp, the driver filed the tank again with 20 gallons of gasoline. The odometer read 57,060. To the nearest tenth, what was the ears average mils-per gallon for theentire trip? Preview: Solution Hide Solution You Submission: Solution Solution: The entire trip was 57,060 — 56,200 = 860 miles. The tank was fll after the driver added the 6 gallons of gas at the beginning. After that, the driver nad to add @ total of 12 + 20 = 32 gallons of gas to have the gas tank completely ile atthe end So, the car used 32 gallons of gas while aveling the 60 mies, That gives an average miles-per-gallon rate of ‘860 miles 860 miles wallen ‘32 gallons 82 gallon gallon pe cd ‘A twelvehour clock loses 1 minute every hour. Suppose it shows the correct time now. What isthe least rumier of hours from now when it wll again show the corect ime? Hide Solution Yur Submission: Solution Solution: The clock will again show the correct time when it has lost total of 12 hours. Every hour the clock loses 1 minute. We want the clock to lose 12 houts, which is 12-60 = 720 minutes. It wil take 720 hours to lose 720 minutes, so i wil take [Fav hows) lose 12 hous ‘As with mary of our other problems, we also could have used conversion factors to solve the problem: 1 hous time 60 minutes lost “Trniute Tort 0? BOO re at = 12-60 ours timeHomer began peeling a pile of 44 potatoes atthe rate of 3 potatoes per minute. Four minutes later, Christen joined him and peek ‘the ate ofS potatoes per minute, When they fished, how many potatoes had Christen peeled? ide Soation Your Submission: Solution ‘Solution: Homer peels @ potatoes a minute, coin the four minutes he works alone, he peels 3-4 = 12 potatoes, This leaves 1 ~ 12 = 82potatoes for hem to peel together. Together, they peel 3+ 5 = 8 potatoes per minute, s0 they need 82 potatoes = 4 minates to peel the rest ofthe potatoes. Christen peels 5 potatoes a minute for 4 minutes, so she peels 3-4 The cold-water faucet of a bath tub can fillthe tub in 1S minutes, The drain, when opened, can emply the ful ub in 20 minutes. Suppose ‘the ub is empty and the faucet and drain are both opened at the same time, How long wil take to fill the tub? 12y ype any additional notes you have he Hide Solution Your Submission: Solution ‘Solution: Since the faucet can fil he tub in 8 minutes, i fille 1 ‘of the tub every minute Silly, the drain empties <1 of the tub B z me ea every minute. Therefor, the fraction ofthe tub tha is filled each minute when the faucet and drain are both open is 11.4 3.1 0-H 1 Since Gg of the tub is filed each minute, wil teke| for the ub to fil, Roger can shovel his family’s weway in 1 hour His older sister Alexis a shovel the driveway in hour: they work together, then how long wilt take them to shovel the driveway? Hide Solution Your Submission: Solution 1 Solution: Since Roger can shovel the driveway in an hour he can shovel 2 ofthe civeway every minute, Sina, Alexis can shove 1 te diveway in 30 minutes so she shovels +f the crveway each minute Togetiey, each minute they shovel 1 30 ofeach i ete aed atta tee nearer PAR pee cd Three water pipes are used to fila ewimming pool. The fist pipe alone takes @ hour to fil the pool, the second pipe alone takes 12 hours toil he pool, and the third pp alone takes 24 hours to fille pool. all tree pipes are opened atthe same time then how long wil take to fille pool? Hint Problems involving rates and work can often be solved by thinking about how much work is done per unit oft‘int: What fraction ofthe poo is filed each hour? Preview Solution You may ype any additional notes you have here Hide Solution vow Stn Sn 1 souson eft ppesisLotepain hate seoncae tL ene petntnocane tee ot npn sh Sot pee Lai saa 6 stiato mt at ai aa ofthe pol in 7 hour Therefore te ve pipes together wil take our] oil te pool7.7 Summary ‘Aatio ia used to compare the relative quantities of two of more groupe or items. However, a ratia only compates the quantities to each ‘other-it doesrit tellus the actual values of the quantities. Far example, suppose that you know that certain history class has a rtio of ‘iis to boys of 2: All this tells you Is that fr every 2g, there are 3 boys. It doesrit tll you how many boys orgs there are. Concept: ratio gives aelative comparison of two quantities. It doesnt tell you anything about a the total amount ofthe quantities, Definition: To simplify a ratio means to write it as a ratio of integers with ne common factor ‘greater than 1. Important: Suppose we are using a ratlo to compare two quantities that together make up @| ° ‘up (such as gts and boys in a class). Ifthe two quantities are in the ratio a:b, tne ais oof nn ne en ye b vo petite whole ‘Whenever we have two ratios that are equal, we have a proportion. The most common usage of proportion is when we have two changing ‘quantities that ae related in such a way that thel ratio doesnt change. ‘A.commenly used rato is speed, which stheratio of distance to time: We can write his as an equation a “This equation oan be rearranged a (speed) - (time) = distance and also a8 distance tine = “The units associated with speed help us remember these equations. The word per essentilly means “dlvided by” soa speed in miles per hour means to take distance (in miles) and vice by time (in hour) WARNING Speeds do not usually “average” inthe way that you might expect them to, @ Speed is just a special example of arate. Whenever a quantity changes by a certain amount in a fixed unt of time, we have a rate, Just as vith speed, the use of the word "per Is often a signal that we are working wth arate. Also just as with speed, we can use the units to our _adirantage when solving problema. Concept: Pay close attention to units in word problems! Use the units In your mathematical & ‘expressions tofielp you figure out how to use the information in the problem. Unit conversions are just ancther type of rate, We can work with ates much like we work with conversion factors. In particular, we use ‘them in mathematical expressions so that the units ezncel in the way that we wantReview Problems The rato of eats to dogs atthe pound is 2: 3. thete are 18 cas, then how many dogs are there? Hide Soation Your Submission: Solution Solution: Muhtplying both parts ofthe ratio by 9 gles cats: dogs a or [27] ge att pe The ratio of boys to gis at 2 summer camp is 4 to 5. f the total numberof stucents atthe camp is 108, then how many boys are a the Preview: Solution ide Soation Your Submission: Solution 4 rea 4 Solution: Since the ratio of boys to gts atthe camp is A: 5 the boys are = = 5 ofthe students at the camp, There are 108 4 students, 0 there ae = +108 = [18 boys. oC acd Given that ene pound Is sixteen ounces, whats the ratio of 7 pound, 4 ounces to 3 pounds, 10 ounces? Preview: Solution Hide Solution Your Submission: Solution Solution: We express both welghts in ounces, so that we can compare them. 1 pound, 4 ounces equals 1- 16 +4 = 20 ounces. 3 pounds, 10 ounces equals 3-16 + 10 = 58 ounces. So, the ratio of the given weights is 20 : 58, which is[10 = 29] in simplest form, ‘Aboard that is 12 meters long is cut into 2 pieces whose lengths havea rato of 1: 5, What isthe length ofthe longer piece? ide Soation Your Submission: Solution Saluon: The longer piece Is == = Fo the whole oud soe eng ofthe longer pce fs & (12 meters) = [0 meters| | The ratio of boys to girs nan assembly is 4t0 3. How many students are pres Hide solution You Submission: Solution pee cd AL end? die spends $5, Andie spends $120 more than Marisa does. How many dollars does Andi For every 3 Marca spen Hide Solution ay ype any addtional notes you have he Your Submission: Solution ‘Solution: The ratio ofthe amount Marisa spends to the amount Andie spends fe 3 dollars and Andie spends Sx dollars. Andie spends $120 more than Marisa spends, so Si: — Sz 22 = 120, so. = Gi. Therefore Andle spends Sir oO eres is © guopy ot an angelfish. The rato of guppies to angelish in the tank ie 8 : 2. Originally, there are 20 fishin a tank, and each fs Twenty more fish are added tothe tank. Each new fsh is ether a guppy ar an angelfsh. The ratio of guppies to angelfish after the fish fre acded i 2:3 How many qupples were added tothe tank? 5. So, for some value of 2, Marisa spends Sr 120, simplifying gives Hide Solution Your Submission: Solution a there are 12 guppies in the tank ‘Solution: nally the fraction of fish that are guppies ig = = h, ouppiesis 55 er ado he new sh here re 20 +20 = 40 fh nthe nk Sire the oof uples to angelishe then 2: 3 we tao iat fale espa tae wath omer] epee mee 2 2 Fay = zo these fich are guppies. So, there 243 bai acide. veo Ce wed $26,000, The four partners in @ business decide to spt the profits of thee company in the ratio 2: 8: 3: 5. the profit one year is then what isthe largest amaunt of prof received by one ofthe four partners? Hide Solution Your Submission: Solution Solution: The person who receives the largest amount gets =" = — of the total profit. So, this person receives e id Serre a8 = &(826,000) =| $10,000 The statue af Abraham Linoaln in the Lincoln Memorial in Washington, DC, ia 6 meters tall. On the back af the $6 bil the statue measures S:milimeters tall. Ifthe Memorial measures 25 mm talon the back ofthe $5 bil, and assuming the bls dawn to scale then how tals the actual Memorial in Washington? Hide Solution Your Submission: Solution Solution: The statue's height and the Memorials helght are In proportion. On the back ofthe $5 bil, the Memorials 25 mm and the statue is mm, go the Memorial is 5 ties as tll asthe statue. Therefore the real Mernoial is 5 times as tall asthe real statue, or 5. (6 motes) = [20 meter al ve ve Ona man, wo moun ae 5 ne pata neh nthe pee il Row many ie pa et wo mountains? Hide Solution You Submission: Solution 160 = 160 = 47-20 =(940 miles Ss 1 a Solution 2 We use conversion factors. Since inch corresponds to SO miles, Inches corresponds to 7 sactes) - 80 miles 5) Finches 50 niles = (2m) ain A fortnight i 14 days, and a mile is 8 furlongs. fa desert caravan travels 10 miles per day, then how many furlonge does it rave! in 2 fortnight? Hide Soation Your Submission: Solution ‘Solution: We set up conversion factors: ales ils 8 furlongs fe gee USS s ‘day T mile = 0-8-1) iene Toctuight furlongs‘A tnch by Sinch photo is enlarged proportionally such that te smaller dimension is now 1 foot 3 inches. How many inches are In the larger dimension? Hide Soation Your Submason Stn {Srp lot mere rere he erga pelo eS 78 Th mir Genel oe ele ge pte 12-43 = 16inches Matphing both pars oF Sy & es 15: 25 ete ger dmerion fhe ptr [25 nes ve eis ‘On my tourist map of Quebec, the distance between the dot representing Montreal and Quebec City Is 10 em. Ifthe scale ofthe map is lay, then how far apart are Montreal and Quebec City? (Note that 10 mm = 1 em.) ide Solution Your Submission: Solution 4 Solution: The rato of the cistance on the map to the actual distance is {min : 1 km. integers are easier to work with than fractions, so we multiply both parts of the ratio by 9 to ge 4 mm : 9 km. Next, we notie thatthe dlstance on the map between Montreal and Quebeo City is n centimeters, not mm, In ofdet to use our ratio, we convert the distance on the map to mm. Since there are 10 mm in every 1 om, 10 em equals 1 Multsying both pars of the ratio d mm : 9 km by 28 gives us 100 mm : 226 kim 80 Monteal and Quebec Cyan part. Dale ravels from oty A to city B to tty and back o cty A. Each fy is 120 miles from the other two. Her average rate from city A to city 1s 60 mph. Her average rate from ely 23 o etty Cts 40 mph. Her average rate from clly Co ely ie 24 mph. What is Dale's average rate forthe ent trp, miles per hour? ide Solution Your Submission: Solution ‘Solution: Dales tp from Ato Bia 120 miles a 60 mph, which takes 120 miles _ 120 = 20 hours = 2 hours 6 mae Hee tip from 130 C ls 120 miles at 40 mph, which takes 120 mil 12 120 tous = 3 hours. Hertrip from C'to A is 120 miles at 24 moh which takes 20) 120 miles 4 Spm = py ts = 5 hours, 4s Combining these, she travels 120 +120 4 120 = 360 miles in 24-345 = 1Ohours, co her average speed is 30 miles _ 360 miles _ ae Thows — 10 hour — | towThe density of tiquld is 8 pounds per gallon, and the density of liquid 3 ls 6 pounds per gallon. What quantity of liquid 2 weigh same as 30 gallons of liquid A? Preview: Solution ype any addtional notes you Hide Solution Your Submission: Solution Solution: 20 gallons of quid A weighs 30-8240 pounds, Since each gallon of liquid Jf weighs 6 pounds, we need 240/6 = [40 gallons] of liquid 7 t0 weigh the same amount We could also set this up using conversion factors as Btbof A 1tbotB 30 gal of A= 30 gal of A puoi . : wloid 1ibofA alot B 40 gal] of B. oa Ld Ike speedometer on his motoreyele is broken. He is ing ata constant speed. He times himself and finds that it takes him 1 minute and 20 seconds toride 1 mile, How fasts Ike ring In miles per hour? Preview: Solution Hide Soation Your Submission: Solution Solution: Siace 1 minute Is 60 seconds, ike covers 1 mile in 80 seconds. We ean use conversion factors to convert "miles per second’ 10 “miles per hour’ mile 60 seconds 60 mines Weseconds Tminite 1 hor 60-60 ile =F a0 four ~ [Be poo) Elsa swims lope in the pool, When she first started, che completed 10 laps In 25 minutes, Now ehe ean finish 12 laps in 24 minutes, By how many minutes has she improved her lp time? Hide Solution Your Submission: Solution ‘Solution: When Elisa started, ok her 28 minutes to cover 1O laps <0 each ap tok her = 5 minutes. Now, se swims 12 aps‘Anorthbound tain from Miami to Jacksonville made the 324-mile joumey at an average speed of 50 miles per hour On its southbound return tip, It made the journey at an average speed of 40 miles per hour. To the nearest tenth of @ mile per hour, what was the trans average speed fr the 642-mile roundtrip journey? ye any adationa notes Hide Solution Your Submission: Solution ‘Solution: The northbound train travels 324 miles at 0 miles pet hour, so is wip takes 8 _ 3 =F house = 6.48 hours b miles = i hours = 8.1 hour Combining these, the tain covers 324 4. 324 = G48 miles inG.48 48.1 = 14.58 hours, for an average speed of mph # [44-4 miles per hon 648 miles TASS hous — var ee cd Boston is 295 miles from New Yor ity along a certain route. A car starts from Boston at 1:00 PM and travels along the route towarel New York at a steady rate of §0 mph. Another car starts from New York at 1:20 PM and travels along this route toward Boston at 2 steady rate of 40 mph, At what time do the cars pass each other? ye any addition Hide Solution Your Submission: Solution Solution: During the fst half hour of ts journey the ear from Boston goes i (3%) 270 miles at a 00 ae 3H forthe dotance ta decrease to 0. Therefore the ears meet a {A seasonal pond in my yard has 1000 gallons of wate. If water evaporates atthe rate of 12.5 gallons per day and no other water fe added or removed then how much water wll be in the pond after 30 days? Hide Solution Your Submission: Solution ‘Solution: In 30 days, the amount of water that evaporates ie gallons ‘ay (20 ans): (12 ) = 375 saloon Therefore the amount of water left after 30 days is 1000 ~ 375 — [625 gallonsMegan has three candles ofthe same length to provide ight. Candle Al burs for exactly 72 minutes. Candle 1 burns twice 2s fast a candle A, Candle C’ bums three times as fast as candle 12. What isthe greatest total number of minutes of light that all hree candles can provide? Hide Solution Your Submission: Solution Solution: Since candle 1 buns twice as fast as candle A, takes hal as long for candle D3 to bum out as candle A. Therefore, 1 candle 1 bums for 5 - (72 minutes) = 36 minutes. Sina, candle C bums for one-third as long as candle 1, so candle C ume for 1. (36 mimtes) = 12 minutes. Therefore, we bum one candle at a time, the candles can give 72496 +12 = [120 minnte] of toh baa ahd Working alone, Jamie can mow her lawn in 75 minutes, If hls he, then the two ean maw the lawn in 30 minutes. Haw long does it take Bob to mow the lav alone? Hide Solution You Submission: Solution Solution: Since Jamie can mow the lawn In 75 minutes, she can mow ze of her lawn each minute, Therefore, In 30 minutes, she 3 ‘of the lawn. This means that Bob mows the other 1 — = = 5 ofthe lawn in 30 minutes. There are several ways we can ins from here. a We could note that f lob mows © ofthe lawn in 30 minutes, then he can mow more Ee 3 We could alao use conversion factors to find hie rate of lawn mowing. He mows 5 minutes | per lawn. 5 ee eres ence ‘ofthe fain in 20 minutes, 20 his rate 30 minutes _ 30 minutes ing on vacation from Mexico to London, witha brlef stop In New York. He forgot to exchange his pesos for Br ‘and must do so In New York. He would lke to have 2000 British pounds for his trip. 12.1 Mexican pesos can be exchanged for 1 dalla ‘and 1 dollar can be exchanged for 0.62 pounds, To the nearest peso, how many pesos will Carlos have to ex order to get 2000, Bish pounds? Hide Solution Your Submission: Solution ‘Solution: Applying conversion factors foreach ofthe exchanges, we have dollar 12.1 pesos 62 pons ~T dollar 2000 19.1 = Aaa — Povo = [BO0TR pesos 2000 posureyeeds to be ready in 18 Four short order cooks can make 24 omelets in 10 minutes. Ifa diner gets a to-go order for 90 omelets th ‘minutes ten how many cooks do they need complete the order on time? Hide Solution Your Submision: Solution ‘Solution: We know that 4 short-order cooks ean make 24 omelets in 10 minutes 1 cook can make as many omelets s 4 cooks can, s0 i 1 hoctorder cook can make 6 omelets in 10 minutes) 153 3 I the cook has 15 minutes instead of 10 minutes, then he has + = 5 times as much time, So, he can make 5 times as man Ifthe cook has 15 ead of 10 minutes, then he has == = 5 th h time, S 3" y es omelets. Since 5 = 9, we have 1 short-order cook can make 9 omelets In 15 minutes 0 Since we need 90 omelets in these 15 minute, and each cook males 9 omelets in 15 minutes, we needChallenge Problems of 45 dimes is cvided inta tree piles in the ratio =: + < =, How many dimes are inthe pile with the least numberof dimes? ® ea . Preview: Solution Hide Solution Your Submission: Solution Solution: The common denominator of the factions is 12, 20. we ean convert all the terms in the ratio to integers by multilying them by 12: 2 Teper tele mao esa 2 cee mes, osha ple hae 2 45 =[0] ames, 3 Teass~ Five workers together can build a read in 20 days. Suppose every worker works at the same rate If tree workers work on the road for 10-days before eleven more workers jon them, then how long total wil take to buld the road? Hint Problems involving rates and work ean often be solved by thinking about how much wrk i done per unit of time Fmt: How long would ita iat fraction ofthe road does one worker complete In one day? ne worker to bulé the whole road? W Proview: Solution 7 Hide Solution Your Submission: Solution Solution: Because § workers ean bullé road in 20 days, each warker can build ~ of a road in 20 daye, Therefore, each worker can inn ati ss 20-5 = 10D cr Sec waa 2 nh Tele wer vet Bl : ya a7 Fe ees i ergo east = gender vier Se en ACh wey 100 10 7 “ 10 i 4 z Alter the 17 extra workers Joi in the 14 workers bull = ofthe road each dey. So, 10 fish the remaining 7 ofa rood, the workers must work for ‘Therefore, the road sult in 10.4.5 oy 7.60: When Paul erossed the finish line of a 6O-meter race, he was ahead of Robert by 10 meters and ahead of Sam by 20 meters. Supnose Robert and Sam continue to race the finish ine withaut changing thei rates of speed, By how many meters will Robert beat Sa? Hint By how many meters would Robert beat Sam i the race were 50 meters? 100 meters? 150 meters? Preview: Solution f Hide SolationYu Submission: Solution Solution: When Paul finishes, Rober has run 60 ~ 10 = 60 meters and Sam has run 60 ~ 20 = 40 meters. Therefore, when 4 ofthe distance that Robert covers. So, while Robert runs the 40 Robert and Sam run for the same amount of time, Sam covers 2 4 55 10 = 8 meters, This means Rober’ lead over Sam increates by 2 more meters and he final 10 meters of the race, Samm rune = [2] eters oA Sn nd WA? = 0116 then what isthe value of x? beats Sam by 10+ Preview: Solution Hide Solution Yur Submission: Solution ‘and multiplying both parts of 2: 16 by 4 gives = dur : 64, 80:2" = 64, which gives x = Solution: Multiplying both parts of 4:2 by x gives 4:27 = de we; 16 = de : 64 Substituting these ino 4: 2* = x : 16 gWes 40 5 sea lhe rte he mont re ean oe noc = My bh ety andy ot reiditadeernaatonl pias diem ea = apse = (A a acd The Big Telescope Company sells circular iors. Thar largest mirrors have rai of S meters and their smallest mirrors have adi of meter. The cost of every mira Is proportional tothe cube of the mir's radius. What Is the ratio ofthe total cost of 25 of the companys smallest mirors ta the cost of one of the company‘ largest mirore? Hide Solution You Submission: Solution ‘Solution: Let the cost of ane of the amallest mirore be $0 the cost of 25 ofthese mirors fs 25. The rads ofthe largest miro is 5 times the radius of the smallest mirror. The cost of the miro is proportional tothe cube of the rads, so the cost ofthe largest mirror is 5? = 125 times the cost of the smallest mirror. Therefor, the cost of the largest mirror is 125i, andthe desired ratio Is 25x: 125, Diving both pars ofthe ratio by 252 gives Kim was elected class president. She recelved 3 votes for every 2 that Amy got. No one else ran. However, 8 ofthe peo for Kim had voted for Amy instead, Kim would have raceived only 1 vote for every 2 that Amy would have gotten, Hew many people voted? Hint: Assign a variable so you can write expressions and set up equations, What's often a useful way to assign 2 variable in a ratio problem? Hide Solution Your Submission: Solution ‘Solution: The rato of Kins votes to Amy's votes Is 3 2,80 Kim recehved Sr votes and Amy received 2s votes for some value of x IF 8 ofthe people had voted for Amy instead of Kim, then Kim would have 82 — 8 votes and Amy would have 22r +8 votes If this hed happened, then Amy would have twice as many votes as Kim, so 22 + 8 = 2(3e ~ 8). Expanding the righthand side gives 2x 48 = Gr ~ 16. Subtractina 2x from both sides and adline 16 to both sides ives 24 = dx. 90 x = 6. Since Be 2r = Sie
You might also like
Art of Problem Solving Middle School
PDF
100% (1)
Art of Problem Solving Middle School
188 pages
Placement Test Geometry
PDF
No ratings yet
Placement Test Geometry
6 pages
Nelson Mathematics 7 Isbn13 9780176269128
PDF
No ratings yet
Nelson Mathematics 7 Isbn13 9780176269128
521 pages
Pre 4A
PDF
No ratings yet
Pre 4A
4 pages
Math League 12429
PDF
No ratings yet
Math League 12429
29 pages
DynamicMathBook-OntarioGrade9-CompleteDe Streamed AlanR Taylor BillKokoskin TylerHuculak Surrey, BritishColumbia, 2019 9781990448331 Anna'SArchive
PDF
100% (1)
DynamicMathBook-OntarioGrade9-CompleteDe Streamed AlanR Taylor BillKokoskin TylerHuculak Surrey, BritishColumbia, 2019 9781990448331 Anna'SArchive
356 pages
Ajinkya Pawar - Sanford 2024-2025 Rawle Orthodontics Scholarship App
PDF
100% (1)
Ajinkya Pawar - Sanford 2024-2025 Rawle Orthodontics Scholarship App
10 pages
Pre 5C
PDF
No ratings yet
Pre 5C
5 pages
Ramanujan Mathematical Society PRMO Etc
PDF
No ratings yet
Ramanujan Mathematical Society PRMO Etc
356 pages
Wrinkle in Time Questiions Chapter 3
PDF
No ratings yet
Wrinkle in Time Questiions Chapter 3
3 pages
Mathcounts Lesson 1
PDF
No ratings yet
Mathcounts Lesson 1
13 pages
Math Count Problems
PDF
100% (2)
Math Count Problems
25 pages
2025 AOPS Practice Sprint Round
PDF
No ratings yet
2025 AOPS Practice Sprint Round
22 pages
2010 Euclid Solution
PDF
No ratings yet
2010 Euclid Solution
16 pages
4d Practice Exc1
PDF
No ratings yet
4d Practice Exc1
4 pages
1 7cambridge Checkpoint Mathematics Coursebook 7 Paperbacknbsped 110764111x 9781
PDF
No ratings yet
1 7cambridge Checkpoint Mathematics Coursebook 7 Paperbacknbsped 110764111x 9781
7 pages
Intro To Number Theory
PDF
No ratings yet
Intro To Number Theory
122 pages
Russian Sharygin Geometry Olympiad (2008-2012)
PDF
No ratings yet
Russian Sharygin Geometry Olympiad (2008-2012)
14 pages
Richard Rusczyk, Sandor Lehoczky - The Art of Problem Solving, Volume 2 - and Beyond Solutions Manual 2 (2006, AoPS Incorporated) - Libgen - Li
PDF
No ratings yet
Richard Rusczyk, Sandor Lehoczky - The Art of Problem Solving, Volume 2 - and Beyond Solutions Manual 2 (2006, AoPS Incorporated) - Libgen - Li
183 pages
Pre 2D
PDF
No ratings yet
Pre 2D
4 pages
Detailed Notes On Ratios For 7th Grade Math
PDF
No ratings yet
Detailed Notes On Ratios For 7th Grade Math
4 pages
2020 State Competition Solutions
PDF
No ratings yet
2020 State Competition Solutions
13 pages
Pre 4B
PDF
No ratings yet
Pre 4B
5 pages
Ratio - Word Problems
PDF
No ratings yet
Ratio - Word Problems
3 pages
MathCounts-2009 Sprint Round (Chapter Competion) - KEY PDF
PDF
100% (1)
MathCounts-2009 Sprint Round (Chapter Competion) - KEY PDF
8 pages
Ractice Ompetition: Sprint Round 1 15 Target Round 1 4 Team Round 1 5 Answer Key Solutions
PDF
100% (1)
Ractice Ompetition: Sprint Round 1 15 Target Round 1 4 Team Round 1 5 Answer Key Solutions
7 pages
Mathcounts Sprint Round 1991-1992
PDF
No ratings yet
Mathcounts Sprint Round 1991-1992
3 pages
MC12 Slide Presentation
PDF
33% (3)
MC12 Slide Presentation
40 pages
PB CH 7 - 221118 - 164520
PDF
No ratings yet
PB CH 7 - 221118 - 164520
28 pages
Hypatia Combined Contest
PDF
100% (1)
Hypatia Combined Contest
25 pages
Part1 QP
PDF
No ratings yet
Part1 QP
12 pages
AoPS Post Test Intermediate Algebra
PDF
No ratings yet
AoPS Post Test Intermediate Algebra
3 pages
Sample Calculus AOPS
PDF
No ratings yet
Sample Calculus AOPS
7 pages
Sample M
PDF
100% (1)
Sample M
3 pages
2019 Chapter Competition Sprint Round PDF
PDF
No ratings yet
2019 Chapter Competition Sprint Round PDF
7 pages
Properties of Arithmetic: 1.1 Why Start With Arithmetic?
PDF
No ratings yet
Properties of Arithmetic: 1.1 Why Start With Arithmetic?
9 pages
2005 State Mathcounts Solutions
PDF
100% (1)
2005 State Mathcounts Solutions
13 pages
Meet 18 PDF
PDF
No ratings yet
Meet 18 PDF
14 pages
MathCounts-2010 Target Round (State Competion) PDF
PDF
No ratings yet
MathCounts-2010 Target Round (State Competion) PDF
5 pages
This Diagram Was Taken Off The Internet
PDF
No ratings yet
This Diagram Was Taken Off The Internet
1 page
Intermediate Algebra Posttest PDF
PDF
No ratings yet
Intermediate Algebra Posttest PDF
3 pages
Are You Ready For: Precalculus
PDF
No ratings yet
Are You Ready For: Precalculus
3 pages
99 Conditional Probability May2020MinisHandout
PDF
No ratings yet
99 Conditional Probability May2020MinisHandout
2 pages
2013 MATHCOUNTS School Target Round
PDF
100% (1)
2013 MATHCOUNTS School Target Round
8 pages
CML Contest
PDF
No ratings yet
CML Contest
16 pages
Spiky Door - A Geometry Project
PDF
No ratings yet
Spiky Door - A Geometry Project
2 pages
Aops Bact12
PDF
No ratings yet
Aops Bact12
14 pages
Advanced Topics in Inequalities - Franklyn Wang Et Al. - AoPS 2015
PDF
No ratings yet
Advanced Topics in Inequalities - Franklyn Wang Et Al. - AoPS 2015
13 pages
2005 National Team
PDF
No ratings yet
2005 National Team
3 pages
All of Math in Three Pages: Individual Test Topics
PDF
No ratings yet
All of Math in Three Pages: Individual Test Topics
3 pages
Aops Mock Test1
PDF
No ratings yet
Aops Mock Test1
2 pages
Complex Number Questions PDF
PDF
No ratings yet
Complex Number Questions PDF
1 page
Art of Problem Solving Information, MMA PDF
PDF
No ratings yet
Art of Problem Solving Information, MMA PDF
6 pages
Chicago Math League Grade 7 2012-13 Solutions
PDF
No ratings yet
Chicago Math League Grade 7 2012-13 Solutions
2 pages
Problems Set II
PDF
No ratings yet
Problems Set II
1 page
5 /6 Grade Math Syllabus Ann Marie May 1. (Before School Starts) Now-End of August
PDF
No ratings yet
5 /6 Grade Math Syllabus Ann Marie May 1. (Before School Starts) Now-End of August
3 pages
11 Whole Numbers: Mental Arithmetic and Estimation
PDF
No ratings yet
11 Whole Numbers: Mental Arithmetic and Estimation
8 pages
2009 Euclid Contest: Canadian Mathematics Competition
PDF
No ratings yet
2009 Euclid Contest: Canadian Mathematics Competition
15 pages